You are on page 1of 74

dd

PEDIATRIC

1
• infantile colic
1-Pt with infantile colic scenario, treatment :
A. Change his milk formula because he might be allergic
B_ Reassure mother that it’s normal " $ start on 6 weeks **
#
N.B. Infantile colic onset is 6 weeks of age and goes away by 6 months of age
‫رﺑط = ﻋددﺣروف‬colic. ‫= ﺧﻣﺳﺔ = زودواواﺣدوﯾﻛوﻧﺳﺗﺎﺳﺎﺑﯾﻊ‬

2-Infantile colic mc occur :


A. First 6 weeks" $**
#
B. First 3 weeks
C. First 3 months
D. First 6 months

3-Case describe infantile colic ... what is the usual duration ?


1st 6 month**

4-consul mother regarding infantile colic ?


A. 80% of children have infantile colic
B. will resolve at 6 month"
$**
#
C. gharib water have evidence in treat infantile colic

5-A child with excessive crying and can't stop crying. There is history of *bottle feeding*. On examination,
the child is restless, and abdomen distended. No other significant finding on examination. Dx?
A. Infantile Colic "$**
#
B. Infant constipation C. Cow milk intolerance

6-What’s true about infantile colic = w large passage of flatus**‫ﯾﻛوﻧﻌﻧدھﻣﻐﺎزاﺗﻛﺛﯾر‬

7-typical symptoms of infantile colic, all normal, what is most appropriate management?
A. Colic drops for colic
B. Changing formula
C. Behavioral adaptation"$**
#

8- a 6 week baby he spilt out whatever he drink he is now on cow milk his weight in birthday : 2,7 kg and
now he is 5.3 kg:
A. observation
B. precautions of reflux and reassure"
$**
#
Keywords : split out after feeding

9-Baby 4 months, distrnded abdomen, stool yellow and it becomes the color more ‫ﻟوﻧﮭﺎﻓﺗﺣﯾوﻣﺑﻌدﯾوم‬
,after birth he passed stool. Dx ?
A- allergy to formula given(formula intolerance )" $* *
#
B- Volvulus
C- Diverticulum inflammatory
Keywords : stool yellow and it become the color more = change of stool color
10-Infant on breastfeeding after that take bottle feeding and complaining from constipation and .distension
dx?
A-Cow milk intolerance" $**
#
B-Infant colick
Keywords : stool yellow << allergy to formula

2
11-Child 12 month ago on breastfeeding for first 9 months then use bottle feeding for another 3 months.
Came with symptoms and signs of anemia with splenomegaly+hypochromic microcytic anemia with
retIculocytes number normal *RBC* count Peripheral blood smear *Target cell* What should be
restored = Iron
Key words : target cell come with iron deficiency anemia and sickle cell anemia , and if there’s reticular cell
normal we go with iron deficiency anemia because when the reticulocyte high means there’s fracking in
blood what cause of fracking of blood ? hemolysis or other cause leading to hemolysis , any causes leading
to hemolysis will increase the reticulocyte , and here reticulocyte normal so we go with iron deficiency
anemia

12-Mother C/O child spitting after each feed cow's milk, Mx?
A- Elevate head during feeding ." $**
#
B- U/S C-Reassure
‫ﺣﺗﯨﺎﻟرﺿﺎﻋﺔاﻟطﺑﯾﻌﯾﺔﻟوﻣﺎرﻓﻌﻧﺎرأﺳﺎﻟطﻔﻠراﺣﯾﺷرﻗﺣﻠﯾﺑﻔﺎاﻟﺣﻠﻧرﻓﻌرأﺳﮭﺎ‬،‫ﻣﺎﻟﮭﻌﻼﻗﮫ‬،‫اﻟﺳؤواﻟﺣﺎوﻟﯾﺧرﺑطﻛوﻛﺗﺑﺣﻠﯾﺑﺎﻟﺑﻘر‬،‫طﻔﻠﺑﻌدﻛﻠرﺿﻌﺔﯾطﻠﻌﺎﻟﺣﻠﯾب‬
‫ﺛﻧﺎءاﻟرﺿﺎﻋﺔ‬
Keywords: spitting after each feed

13-baby with “spitted up” after feeding his weight at birth 3.4 and now 5.5 how should you council the
mother?
let the mother time spent purpine and elevate the head of baby after feeding

14-a 1 month old baby with sob, low grade fever, tachypnea,prolonged expiratory phase and in cxr there is
bilateral infiltration symptoms is the most common to be presented in this condition ?
1.Cough
2.Poor feeding " $**
#
3.Peripheral cyanosis
4.Nasal flaring
‫ اﻋرﻓﻠواﻣﻘﺎﻟﺗﻠﻛﮭذھﺎﻟﻛﻠﻣﺔﻣﻌﻧﺎھﺎاﻟرﺿﯾﻌﻔﯾﮭﺑﻼء‬. ( ‫ﻋﻣوﻣﺎاﯾرﺿﯾﻌﺎذاﺗﻌﺑﺎوﻻوﻟﺷﻲءﯾﺗﺎﺛرﻓﯾﮭﮭو ) ﺗﻘوﻟﻛﺎﻻﻣرﺿﺎﻋﺗﮭﻘﻠﺗﻣﺎﺳﺎرﯾرﺿﻊ‬

15-Child shifted to cow’s milk, pale with wt loss? Iron def.

16-vitamin black color= Iron

17-Feverish child T 38, cough ,Bilateral infiltrated lung. Nothing else mentioned. Management?
A- Reassure cuz viral."$**
#
B- Discharge and oral ABx
C- Admission and IV Abx

18-a 8 month old with asymmetrical breast enlargement and no other symptoms what is the cause =
reassure ‫اقل من سنه مافيه مشكه اكثر يكون‬

19-A child with birth weight 3.5kg now weighing 3.1. Breastfeed about 3 times every 15 minutes. Advise to
mum ?
A. Improve matemal nutrition
B. Reassure that all is well"
$**
#
C. Admit for full work up
Keywords : Weight gain of 112-200 grams a week during the first month. An average 1/2 to 1 kilogram per
month for the first six months. An average of 1/2 kilograms per month from six months to one year
20-Two weeks year old child suddenly had erythematous rash on his body (the child was not febrile and he
was stable) what to do = Assure mom. ‫اسبوعني مافيه مشكله اكثر من اسبوعني نفكر‬
Keywords: two weeks normal
21-Mucus vaginal discharge in newborn baby: reassure the mother

3
22-Child presented to pre-diagnosis clinic with systolic ejection murmur , no sx?
A. refer to cardio for cath
B. reassure and discharge"# $ "**
#$
C.start ACE and
‫ﻣﻌﺎﻟﻣرﺿواﻟﺣرارةﺑﯾﺣﺻل‬murmur
Keywords : innocent murmur its normal

23-Child with 15 months, can only babble and his mother is afraid because he cannot say 2-3 word
sentences, hearing test done when he was 5 months old and was normal:
A. reassure as this is a normal variant"
$**
#
B. re-evaluate at 24 months ‫تم التعديل‬
C. Refer to ENT

24-Child presents with fever, vomiting and diarrhea on exam of chest there is reduced air entry to right side
and murmur was heard , the child CVS and chest exams were previously normal (prior visits) what
is the management of murmur?
A. Urgent echo
B. Reexamine after these symptoms subsides" $**
#
C. Refer to cardio pediatrician

25- 3year are going for dental operation had murmur when he stand and disappear when sitting What to do ?
A. Reassuring"$**
#
B. Reevaluate
C. Cardio consultation
Keywords : innocent murmur

26-Asymptomatic child scheduled for a dental procedure, on examination found to have a murmur that
changes with position, what to do?
A. consult cardio
B. cancel procedure and for further testing
C. innocent murmur ** +‫"ھذااﺳﻣﺎﻟﻣﺎرﻣر‬$no
# need to worry and cont"
$
#

27-Child 3 years old, fell from bed, immediately cried afterwards, vomited twice, headache, physical exam
and neuro exam are normal :
A- Brain MRI
B- Brain CT
D- Observation"$**
#
Keywords : 1) vomting 2 time = observation
2) Vomting 3 time = do brain CT

28-Child was brought to hospital with airway swelling and skin lesions all over the body the mother stated
that he was in a party at his friends house: Food allergy"
$
#

29-If language delay > do hearing test.

30-what to be screen in neonates = metabolic diseases, HYPOTHYROIDISM, HEARING.


Keywords :
In this Q ask about when baby deliver what the first investigation
In other version of Q ask about what first investigation do

4
31-Clear case of congenital prolonged QT syndrome Jervell and Larged-Neilson Syndrome associated with :
sensorineural hearing loss

32- 2years old child can't talk probably and doesn't understand, he have decreased hearing, what 1st test you
want do? Hearing loss examination

33-Baby abdominal distended what is the first investigation = Us"


$**
#

34-Newborn what to examine?


A- Hearing."# $ "**
#$
(: ‫اوﻟﻣﺎﯾوﻟداﻟﺑﯾﺑﯾﻧﻘرءﻋﻠﯨﺎذﻧﮭﻘران‬

35-A child has kernicterus sign what you have expect that you tell his parents ?
ARetina detachment
B.Blindness
C.Hearing loos" $**
#
= ‫رﺑط‬kids = ker‫ = اﻛﺛرﺷﻲءﯾزﻋﺟﻧﺎﻓﯾﮭﻣﮭواﻟﺻوﺗواﻟﺑﻛﺎء‬hearing
Keywords :
Signs & Symptoms
Initial findings associated with kernicterus may vary from case to case, but often include lack of
energy (lethargy) or drowsiness, poor feeding habits, fever, a shrill high-pitched cry, and/or
absence of certain reflexes (e.g., Moro reflex, etc.)

Important photo :

5
36- a 15 month old , parents concerned about his development that only babbles and cant say 2-3 words yet,
he has normal auditory examination, what should you do ?
Reassurance and come at 24 months

37- 3years old boy came with his mother, she's concerned he might have abnomial development. Ho goes to
day care, understand only 2 words command (order) and strangers can understand only 75% of his talk.
Your action?
Delayed speech disorder"$
#

38-Newborn examinations = vision and hearing (vision for red eye reflex )

39-Aspiration meconium treatment = Surfctant nitric oxide if there is problem between o2


Keywords:

Patient post term, was delivered with meconium aspiration, after delivery
was intubated and suction was done, was put on inotropes . After 2 days
now he becomes hypoxic with increased oxygen neads. Pre ductal o2 is 92
and post ductal o2 is 81. What to give?
A. Mgso4
B. inhaled nitric oxide
C. surfactant lavage
D. urgent cardiac catheterization
Answer is B

40-mom comes to your clinic with her neonate who is 1 week of age, she is worried because her neonate lost
1.5 kg from his birth weight? Reassure

41-Infant with erythematous macules on erythematous base in the back and trunk? Reassure
Keywards : erythematous only without fever and asymptomatic >>> just Reassure

42-case of watery diarrhea ( what is the electricity abnormalities will find )?


HYPONATREMIA, HYPOKALEMIA, METABOLIC ACIDOSIS
‫رﺑط= اﺳﮭﺎل = اﺳﯾد = ﻓﻘداﻟﻛﺗروﻻﯾﺗﻛﺛﯾرةﺑﺎﻻﺳﮭﺎل‬
Keywords : Diarrhea = metabolic acidosis alkalosis in urine
Vomiting = metabolic alkalosis flat T wave acidotic in urine
43-Primimother worried about her 6week baby , he stepping all the formula since birth , what most likely:
A. HSD
B. pyloric stenosis
C. physiological reflex"
$**
#
Keywords: 1)stepping is different than vomtining
2) Primimother >>> she become mother for first time its vey important thing
2)first DDx is physiological GERD = stepping
3) second DDx when in Q >>> diagnose by US , olive shape in the abdomen , non bilious vomiting >>>
pyloric stenosis

44-And there was another Q near to the previous, ask about the next step:
A. burping the baby and semi setting while feeding"
$* *
#
B. immediate call surgeon

45-9 year boy with growth pains, management? Reassurance

6
46-Child patient after eat steak has fever and abdomen pain what is the management?
A-Conservative" $**
#
B- antibiotic
Keywords : be focus on duration not give antibiotics , antibiotic is according to duration
Food poisoning ;
1)Food poisoning >>> come with vomiting and may come with diarrhea and may come without diarrhea ,
and sometime occur after 6 hours after eating , and common organism is staph aureus
2)mayonnaise and eggs and cream >>> organism is staph aureus , but
3) rice and ‫ >>> اﻷﻛل اﻟﻣﺣﻣر‬organism is Bacillus cereus
4)meat >>> organism is salmonella and come with watery diarrhea
5)Bloody diarrhea come with Shigella dysenteriae

47-3years old presented with watery diarrhea,cramps , dehydration after being exposed to colleague with
same presentation at day care , mx: === Supportive treatment
‫اﻟواﺿﺣﺎﻧﮭﻔﺎﯾرﺳﻔﺧﻼﺻﺑس‬،‫ﯾﻌﻧﯾطﻔﻠﺗﻌرﺿﻠﻠﻌدوﯨﻼﻧزﻣﯾﻠﮭﻛﺎﻧﻌﻧدھﺎﺳﮭﺎﻟﻔﺎﻻﺧﺗﯾﺎراﺗﯾﻘوﻟﻛﻛﻼﻧواﻋﺎﻟﻣﺿﺎداﺗﺎﻟﺣﯾوﯾﺔوﻻﺑﺳﻌﻼﺟدﻋم‬supportive
care
Keywords: Food poisoning ;
1)Food poisoning >>> come with vomiting and may come with diarrhea and may come without diarrhea ,
and sometime occur after 6 hours after eating , and common organism is staph aureus
2)mayonnaise and eggs and cream >>> organism is staph aureus , but
3) rice and ‫ >>> اﻷﻛل اﻟﻣﺣﻣر‬organism is Bacillus cereus
4)meat >>> organism is salmonella and come with watery diarrhea
5)Bloody diarrhea come with Shigella dysenteriae

48 - 4 year-old child presented with several episodes of passage of frequent non-bloody, watery stool and
vomiting. Management?
A. Metronidazole
B. Doxycycline
C. Amoxicillin
D. Supportive management"# $ "**
#$
Keywords : Diagnosis >>> viral gastroenteritis

49-Child with Hx of eating in a restaurant with abd pain, nausea and vomiting, 10 days later bloody
diarrhea, urinalysis shows 10 RBCs , ttt? hemolytic uremic syndrome
A. Anti diarrhea medication .
B. Metronidazole .
C. Supportive therapy" $**
#
D. Antibiotics

50-Child develops petechia in the legs, he has history of URTI. Labs showed: normal CBC except platelet
count was : low. What is the next step in management? ITP
A. Platelet transfusion
B. Immunoglobulin ( if sever bleeding like untracranial hemhhrage
C. Steroid ( if mild bleeding )
D. Supportive ( if plattlet more than 30.000 and there is no bleeding )"#
$ "
#$
Keywords :
you answer depend on number of platelet in question ,
according the number of platelet in the question you will select the answer

51-Child develops petechia in the legs, he has history of URTI. Labs showed: normal CBC except platelet
count was : low. What is the next step = Supportive

7
52-Child with watery diarrhea , history of sick contact =Supportive management
Keywords :
Is watery diarrhea and no fever

53-5 years old boy has RUQ pain + normal bowel sounds ,lap show severe drop in hemoglobin and
increased reticulocytes count. Blood smear report: target cells and inclusion bodies. diagnosis ?
Sickle cell disease ( Not appendicitis)
Keywords :
Increased reticulocytes

54-Child with a long history of watery diarrhea abdominal bloating and pain, what’s the Dx ?
A-chronic giardiasis✅

55-treatment of dengue fever? Supportive and avoid NSAIDs

56-Dengue mosquito time of activity?early morning"


$
#

57- 2-3years old presented with watery diarrhea,cramps , dehydration after being exposed to colleague with
same presentation at day care , mx:
A-Ciproflaxacin
B-Metronidazole.
C-Supportive treatment"$
#

58-Rota virus: = reassure

59-Child who is lethargic and losing his concentration, Hgb is 10.5, what to give?
&
A- IM iron$
% B- Oral ferrous sulphate. C- Forti cereal. D- Vit,b12
NoteNEVER
: USED

Frist IV iron

60-Neonate in first routen visit had low hemoglobin 10.5 and give ora ferrous sulfate, in this visit also Iow hg
10.3, what next investigation to reach dx?
A. level of serum iron and serum ferritin" $
#
B. bone marrow
C. give it oral ferrous sulfate.
Keywords:
Stable hemoglobin even with ttt find the case if IDA .
‫اﻟﮭﯿﻤﻮﺟﻠﻮﺑﯿﻦ ﺛﺎﺑﺖ ﻣﻊ اﻟﻌﻼج أدور ﻟﺴﺒﺐ واﺷﻮف اﻟﺤﺪﯾﺪ‬

61-Breast milk =: rich in protein

62-Toddler with pigmentations in his teeth, =Sleep with milk bottlecolostrum high in?
A.Protein "
$B.fat
# ‫رﺑط= ﻛوﻟوﺳﺗرام=ﻛوﻟوﺳﺗروﻟﻌﺎﻟﻲ=ﻧﻌﻣﻠرﺟﯾﻣﺑروﺗﯾن‬

63-9 days newbon come with jaundice only in the face not extended to the rest of
the body.. otherwise he is healthy was delivered by NVD with no completing. and
he was breastfeed immediately. what is the cause of his jaundice?
A. Breast milk jaundice."#
$ "
#$
Keywords :
jaundice only in the face not extended to the rest of the body
8
Note :
- Physiological jaundice appear after 3 days of delivery and may continue for 2 weeks 14 days , but this jaundice
appear due to depend of baby on breast milk and sometime beast Malik make jaundice ( unknown cause of
breast Malik jaundice )
- Physiological jaundice 24 - 36 hour
- Breast jaundice first 4 -7
- Breast feeding jaundice 3rd day of life due to insufficient producing breast milk

64-Baby with tooth discoloration, he sleep with bottle at night What the cause?
Bottle dental caries

65-8 yr boy wet his bed at night, the cause is?


A. detrusor muscle."#
$ "
#$
B. Pelvic muscles
- 8 years well toilet trained come with nocturnal enuresis which muscle responsible? Detruser
Keywords:
The detrusor muscle, also detrusor urinae muscle, muscularis propria of the urinary bladder and (less
precise) muscularis propria, is smooth muscle found in the wall of the bladder. The detrusor muscle remains
relaxed to allow the bladder to store urine, and contracts during urination to release urine.
= ‫رﺑط‬Wet‫ﻣﻊ‬Det

66-Night bed wetting considered normal till ?


5 yeaes
‫ ﯾﻛون‬bed wetting‫ ﺳﻧوات اﻻطﻔﺎل ﻋﻧد اﻟﻠﯾل ﻓﻲ‬٥ ‫؟اﻟﻌﻣر‬،‫ﻣرﺿﯾﮫ ﻓﺣوﺻﺎت ﯾﺣﺗﺎج ھﻧﺎ اﻟﻠﯾل ﻓﻲ ﻧﻔﺳﮫ ﺑﻠل ﻣﺎزاﻟﻲ ﻟو ﺑﻌدھﺎ طﺑﯾﻌﻲ‬

67- at what age nocturnal enuresis concerns you ?


-5
-6 "
$
#
-7
-8
Keywords:
Normal till age of 5 you are conceded at age of 6

68-most important study to do in cases of enuresis in child:


a.Urine analysis"$
#
b. urine culture
c. VCUG
Keywords:?
1- renal bladder us
2-vcug

69- Most worrying sign of Child abdominal pain ?


A. peri umbilical
B. Pain for more than 10 min
C. Early morning pain
D. Late night pain
Answer is: D

9
70-A baby girl complains of dehydration and clitorymegaly ,signs of dehydation. next step :
Steriod IVF
Keywords:
Mineralocorticoid is given in classic case of conginital adrenal hyperplasia

71-Old male present with acute confusional state and postural hypotension .. investigation just
hypercalcemia what is the next ??
Hydrocortisone IVF
• Heart disease
72- A young boy complains of arthritis , rashes , nodule subcutaneous.He had pharyngitis two weeks back (
case of rheumatic fever ). What is the most important next step:
A) pedinsone ( very limited rule for steroid )
B) Echo "#
$ "
#$ RF 2 major or one major and 2 minor. Ttt : 1rst ttt
penicilin
C) forgot the opation :( but not related
D) forgot the option :( but not related

73-Boy collapsed during sport On Ex Jerky carotid pulse. what’s Dx?


& $
A. HCOM$%
%&
Keywords:
Hypertrophic obstructive cardiomyopathy.
Significant cause sudden cardiac death in young people

74-Pt came with HCOM Waht tx = Metoprolol


Keywords:
Treatment of HCOM
1- beta blocker
2_ ca channel blocker
3.medication to control arrythmia
4 antibiotic to avoid endocarditis

10
71-child with radiofemoral delay pulse, ?
Coartication of Aorta

72-Baby will be prepared for open heart surgery, mother is very worry about him what is the best way to
deal with her worry?explain what will happen before and after the surgery"
$
#

73-What is Drug decreases the mortality in HF = ACEI/ARB + beta block

74-Management of pericarditis = NSAID like Ibuprofen

75-Sharp pain relieved by leaning forward, pericardial friction rub ==pericarditis

76-PT pos-inferior MI, few hours developed hypotension, raised JVP, clears lungs on auscultation ?
Right ventricular infarction
Keywords:
When you see posterior infarction with hypotension =right ventricular infarction
First degree‫ = ﯾﻛوﻧﺎل‬PR pronlong just.
Scond degree 1‫ = ﯾﻛون‬PR Irregular
Scond degree 2‫ = ﯾﻛون‬rythm ‫ﻏﯾرﻣﻧﺗظم‬
2:1heart block‫= ﯾﻛوﻧﺎل‬QRS = ‫ﻏﯾرﻣوﺟود‬Absent
77-Heart failure due to left ventricular hypertrophy ?
A. systolic dysfunction
B. diastolic dysfunction "
$
#
C. septal defect
D. valve

78-Pediatric patient with sob and productive cough with white sputum for one week that became yellow 3
days ago, on examination there's bilateral crackles increased in the right middle lobe. On x ray report there's
bilateral pleural effusion withconsolidation in right middle lobe. What is the most appropriate cause?:
&
A. Excabitation of heart failure$
%
B. Strept pneumonie
Keywords: ?
middle lobe consolidation goes with respiratory cause

79-2months old baby with congestive heart failure and the mother asking about nutrition requir
11
Greater than requirement for healthy baby"$
#
Keywords:
cardiac pt has to eat more than normal baby

80-Congestive heart failure due to systolic left ventricular hypertrophy give = Beta block +diurtic

81- 14M infant with had recurrent syncopal attacks worsened the HF and i think FTT on examination she has diffuse
crepitations on ECG she has cardiac arrhythmia narrow complex QRS and Hr 250-300 Bpm what is the most probable
cause:
Atrial fibrillation
Atrial flutter
Ventricular tachy
R
Supraventicular arrhythmiaP
Q
Keywords:
250 to 300 b/min while its regular its supraventricular , if irregular its AF or flatter

• TOF
82-Baby after every feeding develop apnea and loss of conscious, what is the cause of admission in
hospital?
A.Syncope"#
$ "
#$

83-children with cyanosis and systolic ejection murmur best heard at the left sternal border. Dx?
Tetralogy of fallot
Keywords:
TOF present after 1 to 6months of delivery PRESENT WITH MURMUR AND BOOT SHAPE HEART
TGA (TPOGA) presents 1 to 6days after delivary NO MURMUR AND EGG shape complication of DM mother
Ttt o2,BBC,PGE1 last surgery
TOF =
1 -pulmonary stenosis
2 - vsd
3 - overriding aorta
4 - right ventricular hypotrophy

VSD = low pitch systolic


ASD = ejection systolic and its fixed splitting S2
COA = systolic murmur at anterior scapular
PDA and TOGV = machinary murmur
tricuspid atresia = pancystol8c murmur
TOF = comes with VSD murmur or systolic murmor and many

to make it more easy to remember add this ?


VSD you should wait till 12 months it could closed spontaneously
PDA can close alone within 2 to 30days
TOF cant be corrected before age of 4years

84-cyanosis with feeding + Holosystolic murmur= TOF

85- TOF 5 yo kid with history of surgical correction at 6 months. Now came with new left parasternal
decrescendo diastolic murmur with single S1 with left parasternal impulse, no radiation. What is the dx?
A.Mitral stenosis
12
B.pulmonary regurg "
$
#
C.tricuspid regurg

86-case of TOF . How does it appear on ECG? right axis deviation

87-Pt with Hx of TOF repair present with murmur in parasternal area it increase with inspiration =
pulmonary regurgitation, becz TOF has pulmonary stenosis.

88-TOF msnag by = pain relif and sedation


Keywords:
Ttt o2,BBC,PGE1 last surgery

89-classic case of DiGeorge syndrome asking about the heart abnormality :


A.TOF "$B.PDA C. TGA D. Endocardium cushion defect
#
= ‫رﺑط‬TOF‫ﻓوت = ﯾطﻠﻊ ﻧﻌﻛﺳﮭﺎ‬Gerog = ‫ = ﺟري‬Googl‫ ﻟل ﺷوز ﻋن ﻓﯾﮭﺎ ﻧﺑﺣث‬. ‫ﻛوﯾس‬foot.

90-Child with peripheral and central cyanosis? TOF


‫ ﯾﻛون واﻟﻔم اﻻطراف ﻓﻲ ﺟﺳﻣﮫ ﻛل ﯾﺧﻠﻲ اﻧﮫ ﻛﺎﻓﯾﺔ ﺑﺎﻟﻘﻠب ﺗﺷوھﺎت ارﺑﻊ رﺑﺎﻋﻲ = ﻓﻣﻧطﻘﻲ ﺗﯾراﺗوﻟوﺟﻲ = ﯾﻌﻧﻲ اﺳﻣﮭﺎ ﻣن‬. ‫ازرق‬

91-Neonate developed cyanosis (2nd or 3rd week after delivery) ry management ?


A. NSAID
B. Prostaglandin" $
#
Keywords:
prostaglandin which is used as a medication. In babies with congenital heart defects, it is used by slow
injection into a vein to open the ductus arteriosus until surgery can be carried out.

91-child with transposition of great vessels. Further evaluation of his mother could reveal ?

A. Prenatal lithium intake


B. Elevated serum TSH
C. Prenatal phenytoin intake
D. Elevated fasting blood glucos " $
#
Answer is : D
Keyword: transposition of the great arteries is called a cyanotic congenital heart defect. Although some
factors, such as genetics, rubella or other viral illnesses during pregnancy, maternal age over 40, or maternal
diabetes, may increase the risk of this condition

92-concern sign (croup ) ? cyanosis

93-Concerning symptom in croup?


a.Blue lips "
$
#
b.Tachypnea.
c.Expiratory stridor
13
94-croup given epi what's next step?
steroids "
$(no epinphron on choic)
#
keyword: start 2 times epinphrin if it doesn’t work steroid
croup = steroid + parking cough

95-Pt congenital heart disease and discretion in x ray egg shape what diagnosis ?
Transpostion *G*reat arterios
E*g*g = *G*reat
Egg = TGA
BOOT shape = Fallot

96-Neonate developed cyanosis (2nd or 3 rd week after delivery) + there is finding on on auscultation
machinery murmur = PDA

‫رﺑط= ﺑدء‬PDA‫ =ﻣﺎﺷﯾن‬machin . ‫ﯾﻼاﻟﻣﺣﺎﺿرةﺑدﺋت = ﻣﺎﺷﯾﯾﻧﺎﻻﻧﻠﻼﻣﺣﺎﺿرة‬،‫= ﻣﺎﺷﯾﯾن‬

97-Child crying and cyanotic= sedation and relieve the pain

98-baby cyanosed with parasternal heave no murmur what to give? Prostaglandin

99-Child pt cyanotlc . can't complete one sentence= intubation


Keyword:Cant one Complete sentence = sever asthma

100-Child, k/c CHD , presented with cyanosis , progresing, o2 sat 85, looks ill, cyanosed, crying
A. immediate catheterization
B. diuretic IV
C. sedation + analgesia"
$
#
Keyword:Diagnosis = TOF

101-2months old baby mother complaining of apnea usually happens after feeding with 10 mins of cyanosis,
why will u admit this case?
A. acute life threatening event"
$
#
B. Seizures C. sepsis

102-Baby born at 27 weeks GA developed SOB, tachypnoea. No X-ray. Diagnosis? apnea of premature
‫واﻟرﺋﺔﺑﮭذااﻻﺳﺑوﻋﻠﺳﺔﻣﺎﯾﻛوﻧﺎﻛﺗﻣﻠﻧﻣوھﺎ‬،‫ﻻﻧوﻟدﺑدري‬

103-Mother brought her 2 years old child to the ER with a history of upper respiratory tract infection for the
last 3 days with mild respiratory distress. This evening the child started to have hard barking cough with
respiratory distress. Which of the following are the most worrisome signs in this condition.
& $
A. Nasal flaring B. Barking cough Cyanosis$%
%&
Croup syndrome = barking cough

14
104-Neonate cyanotic, low o2, Dx:
A. hypoxic"
$
#
B.Hypocapnic
C.Hyoercapnic.
‫= ﻣﻧﺎﺳﻣﮭﺎازرﻗﻠوﻧﮭﻼﻧﻣﺎﻓﯾﺎوﻛﺳﺟﯾن‬

105-What is the least physical activity duration required in pediatrics: 60 min


‫ﺣﺗﯨﺎﻻطﻔﺎﻟﻠﻠزﻣﯾﻣﺷواﺳﺎﻋﺔﺑﺎﻟﯾوﻣوﯾﻌﻣﻠوارﯾﺎﺿﺔ‬
Keyword: children = 60 min per day
Adult = 150 min per week means 30 min per 5 days

106-Patient playing sports and frequently developing sudden attacks of LOC,


examination: mid-systolic murmur in left side, What’s dx ?
Hypertrophic cardiomyopathy

107-Pediatric with holosystolic murmur in left 3rd intercostal space = VSD


3 ‫اﺳد= ﺛﻼﺛﺣروف‬ rd‫ = اذاﺷوﻓﻧﺎﻧﺎﺧذﺷﮭﯾﻘﻘوي‬Holo
VSD : systolic ejection murmur
ASD: systolic ejection murmur
Holosystolic = pan systolic

108- a 8 year , with late systole ,mid sternum ,crescendo decrescendo, high pitched : systolic regurgitation
Click + Ejection systolic murmur +scond intercostal + left = pulmonary stenosis

109-Pistol shot murmur in pediatrics case : aortic regurgitation


‫ﺷوت = ﻗوي = اورطﺎ‬
110-VSD picture , symptomatic = refer for surgery
Keyword: ASD = systolic ejection murmur
111-child with mother death + he is not concentrantion in his lecture = dissociation
‫ﺑﻌدﻣﺎﺗوﻓﺗﺎﻣﮭﺟﻠﺳﺳﺎااﻛﺗوﺳرﺣﺎﻧﺎن‬،‫رﺑط = داﯾﺳوﻛﺎﺷن = داي = ﻣوﺗﺳوﻛﺎﺷﯾن = ﺳﻛوووت‬

• laryngomalacia
112-Child with noisy breathing in prone position decreased when the child is sitting(squatting position), the
mother is worried
A. tell her his condition will get worse B. -He needs surgical intervention
C. He will get better spontaneously at the age of 1 year old"#
$ "
#$
Keyword; diagnosis is Fallot
15
:Squat position

113-Pediatric male pt presented with sign of “ laryngomalacia “ what’s the TTT


A- Referral to surgical departmet
B- reassure and wait for 1 year "#
$ "
#$

114-Infant with airway obstruction that Increase with supine, decrease with prone = Laryngomalacia
$
#
"
‫رﺑط = ﺳوﺑﺎﯾن = زاﯾد = ﯾزﯾد‬

115-baby has breathing sounds his age is 3 months mother is worried


a. normal
b. will outgrow it at age 1 year 90% resolve with time"
$
#
N.B. laryngomalacia

116-Baby crying when change position decreased what couse?


laryngomalacia"
$
#

117-3 months old mother's complain of noisy breathing sounds during sleep and disappeared when he prone
position and increased in supine position What is diagnosis?
A. Nasal atria
B. Laryngomalacia" $
#

118-Baby developed SOB , whezzing DR start to give broncodilator then no response, think of
laryngomalacia dx test is ? Bronchocsopy"
$
#
If laryngocoscope choose it first

119-Laryngomalacia case and ask about the best investigation ? laryngoscope"


$
#

120-tracheomalacia on children = What is diagnostic image ? By Bronchocsopy" $


#
‫ھذااﻟﻣرﺿﯾﻛوﻧﺑﺎﻻطﻔﺎﻟﯾﺗﻣﯾزب‬wheezing‫ﻣﺗﻛرروھواﻧﮭﻠﻣﺎﯾﺎﺧذﻧﻔﺳﺎﻟﻘﺻﺑﺔاﻟﮭواﺋﯾﺔﺗﺗوﺳﻌوﻟﻣﺎﯾطﻠﻌﺎﻟﮭواءﺑﺎﻟزﻓﯾرﺗﺗوﺳﻌﺎﯾﺿﺎاﻟﻘﺻﺑﺔاﻟﮭواﺋﯾﺔﻟﻛﻧﻣﻌﺎل‬
‫ﻧرض‬

121-barking cough=Laryngotracheobronchitis"
$
#

122-Physical exam findings (auscultation) in croup ( patient was having nasal congestion , barking cough) :
A-inspiratory wheeze and prolonged expiratory phase"# $ "
#$
B-inspiratory sounds die subglottic obstruction

123-barking cough = stridor = croup = inspiratory wheeze"


$ .
#

124-A child with inspiratory stridor, barking cough, most likely diagnosis?
-laryngotracheobronchitis( croup)"$
#

125-croup ++ barking cough = inspiratory wheeze and prolonged expiratory phase"


$
# ????
Croup =inspiratoty

126-barking cough and respiratory distress: croup"


$
#
‫رﺑط = ﻛروب‬croup‫ = ﻛرب = ﻟﻣﺎﯾﻛوﻧﺎﻟواﺣدﻓﯾﻛرﺑﯾﺣﺗﺎﺟﺑرﯾﻛراﺣﺔ‬bark‫= ﺑرﯾك‬

127-Barking cough ( croup ) what will listion on auscultation ?


Increase inspirantion sound due to subglottic narrow ( inspirantion stridor ) (Not expirantion à)
16
‫"اﻏﻠﺑﺎﻟﻛروﺑﺳﺎرﺗداﺧﻠﯾﺔﻧﻔﺳﯾﺔ‬
$
#

128-Barking cough ( croup ) what will give ?


Inhaltion epiniphrin and oral steroid"
$
#
.‫ﻛروب = ﯾﺣﺗﺎﺟﻧﻔس‬
Keyword:Epinephrin 2 times - dexamethasone- intubation(last one )

129-Croup classical case barking cough + inspiratory stridor + Auscultation: inspiratory stridor DX : croup
cause : parainfluenza"
$
#
‫ﻛروب = ﻧطﻠﻌﺑرة‬para‫ﻧﺷﻣﺷوﯾﺔﺟو‬

130-child is having barking cough = viral croup"


$
#

131--18 months old child known case of eczema. His parents said he woke up at night and coughed for a
while with barking cough. No upper respiratory tract infection symptoms. Then the cough subsided. A
similar episode happened 6 months ago. What is the diagnosis? 
A. Spasmodic croup 
B. Asthma" $
#

132-Kid with inspiratory stridor, mild respiratory distress, hoarseness of voice, barking cough which of the
following is concerning symptoms - :
A. Tachypnea B. Expiratory stridor .
C. Nasal flaring D. Blue lips"
$ "
# $
#

133-hoarseness of voice in Croup ?


inspiratory stridor with subglottic stenosis (Not expirantion)"
$
#
‫رﺑط = ﻛروب‬croup‫= ﯾﺣﺗﺎﺟﺎﺧذﺷﮭﯾﯾﯾﻘﻌﻣﯾﯾﯾﻘﺣﺗﯨﺎرﻓﺣﻌﻧﻧﻔﺳﯾواﻗﻠﻠﺷدةاﻟﻛرب‬

134-CROUP given epinephrine and after 30 min the symptoms came agine = manag by = Reapate again "
$
#

135-croup given epi what's next step? steroids "


$(no epinphron on choic)
#

136-Croup----------steeple sign on X-ray"


$
#

‫ﻛروب = ﻛرب = ﻻزﻣﺗﻛوﻧﺻﺎﺑر‬steepl

Steeple sign croup

137-Fever, Severe sore throat, high-pitched sound when breathing in (stridor), Difficult and painful
swallowing, Drooling, Anxious, restless child . Feeling better when sitting up or leaning forward diagnosis
??
17
A. Epiglottis"
$.
# intubate
B. bronchiolitis.
C. Pharyngitis
Keyword: drooling +painful swelling

138-drolling saliva= -EPiglottis"


$
#
= ‫رﺑط‬saliva = ‫ = اﺑﻠﻌﻲ‬Epi
Epiglottis-----------X-ray thumb sign‫ﻋﯾب‬c
fEpi
e
d gi ‫= اﻻﺻﺑﻊ‬
h

139-Child with fever ,sob , drooling what next = intubantion and mutiblspichil team = Epiglotitis"
$ ( (
#
Dont choice x ray, bez he ask nest step we will not wait the x.ray))
Yes true treatment intubation because of obstructing in airway
Epiglotitos/ intubation

140-Child preschool age has VSD 2mm,asymptotic,what will you do= Watchful and waiting" $
#
٢ ‫ﻻن‬mm 25 ‫اذااﻟﺛﻘﺑﻛﺑرﻋن‬،‫ﯾﻌﻧﯾﺻﻐﯾرةوﻻﻧﻣﺎﻓﯾﺎﻋراﺿﺎذاﻧﻘوﻟﻧﻧﺗظرﻋﻠﯾﮫ‬mm.‫اﻻھﻣﮭوﻟوﻓﯾﮭﺎﻋراﺿﻼزﻣﻌﻣﻠﯾﺔ‬

141-child have vsd 2mm asymptotic == watchful waiting


.‫"ﺻﻐﯾرةواﻗﻠﻣﻧواﺣدﺳﻣﻔﻣﺎﻧﺳوﯾﻠﮭﻣﻌﻣﻠﯾﺔوﻻﺷﻲءوﺧﺎﺻﺔﻣﺎﻓﯾﺎﻋراض‬
$
#

142-Infant mouconim aspirations management :


$ A-Nitric oxide B-lavage surfactant
#
"

143-child with erythema marginatum , knee pain , fever = what inv =echo"
$
#

144-erythema marginatum = its rash with Rheumatic fever ,we order echo to see any muse heart damage"
$
#

• Virus infection :
• Infection disease 5 type
• 1-meselsse
• Fever- rash criosa
• Most importsnte signs is koplik spot (strt in face and end in leg)
• Cause : paramoxo virus
• Ttt: supportive
• 2-rubela:
• Fever- occipital rash – LN – arthtritis
• 3-Erythema infectiousm (5th disease) :
• Rash in all the body – slap cheke
• Cause : b19 paravirus
• 4- roseola infantum(6h disease)
• Cause : herps v
• Start with fever for 3 day then recovery and start rash
• 5-infectious mononeuclosis:
• Fever- rash- LN- thre thorat – pharangitis – splenomegaly

18
145-Rash on the face and inner cheeck there’s whits spots: = Measles"
$
#
= ‫رﺑط = ﻣوزﻟﻣﺎﻧﻔﺗﺣﮭوﻧﺎﻛﻠﮭﻠوﻧﮭﺎﺑﯾض‬wight spot‫ﻓﯾﺎﻟﻔﮭم‬

19
146-Pediatric patient with coryza, conjunctivitis, and white spots in the mouth, what is the diagnosis?
A-Measles"#
$ "
#$
Measles should mention the 3Cs (Cough coryza conjunctivitis) mouth koplick spots
Keyword: coryza-conj.-spots= measles

Rubella rash starts from the face

Measles neck > behind the ears

147-Child with sore throat and coryza 2 days ago came with difficulty swallow food what is investigation
you will order?
A. Chest x ray
B. Ct scan neck
C. Lateral neck x ray"
$
#
Note:Epiglottitis

20
148-Child with fever conjunctivitis, coryza cough, wheezing Tachypenic what is the optimal ttt
IntubationIntubation "
$ (not Steroid)
#
Note :Epiglottitis
Keyword: diagnosis epiglottitis
149-Young girl has a recent history of fever, difficulty in swallowing solid food only. enlarged 2cm cervical
LN. investigations will you do ? Lateral neck X RAY.

150-Fever and cough and maculopapular rash in behind ear and face and trunk
A. mubes B. measala $
&C. rubella
%

151-7 y/o unvacclnated boy presents with red erythematous irregular patches of rash that is around hls neck
and spreads down hls back. What does he have ?
&
A. measles.$
%
Keyword:
Rubella rash starts from the face
Measles neck > behind the ears

152-A child with runny nose and fever which subsides and then rash appear allover
his body starting from the face. Dx? Rubella, after fever subside it’s roseola, concurrent fever Rubella"
$ .
#

153-Baby with white eye reflex (Leukocoria) and murmur. Mother mentioned viral infection during
pregnancy :
A. Rubella "$
#
B. CMV
C. Toxoplasmosis
. ‫رﻋب = ﻋﯾوﻧﮭﺑﯾﺿﺎ‬
‫ﻣﺷﻛﻠﮫ ف اﻟﻌﯾن وﻋدوي ف اﻟﺣﻣل‬

154-Absent red reflex caused by infection= Rubella"


$
#
21
155-Newborn with absent red eye reflex and new murmur , what does his mother had when she was
pregnant?
Rubella " $
#
Cmv
Toxoplasmosis
‫ﻣﺷﻛﻠﮫ ف اﻟﻌﯾن وﻋدوي ف اﻟﺣﻣل‬

156-rash was red and mother tell it start on face the go to trunk , with LN enlarge of groin= Rubella"$
#
‫رﺑط = رﻋب‬rubella‫واﺣدﺟﺳﻣﮭﻣﻧوﺟﮭﮭﻠﺟﺳﻣﮭﺎﺣﻣر‬

157-Kawasaki management ?
intravenous gamaglubulines
intravenous immunoglobulin (IVIg)"
$ +can treat include: Immune deficiencies like immune
#
thrombocytopenia.

-management of Kawasaki?
A- Aspirin"
$
#
No IVIG in choice

158-Kawasaki sign?
bilateral red eyes "
$
#

159-Case about kawasaki how you will assess the coronary artery disease?
A. iIVIG+ASA
B.ECHO" $
#

22
160-Kawasaki disease assess for heart complication?
Echo"
$
#

161-kawasaki case how you will assess the coronary artery disease?
by 2D echocardiography (2DE) or coronary angiography" $ .
#

162-A child presented with 5 days of fever, oral mucosal lesions, cervical lymph node enlargement and limb
edema. Lab results essentially normal. Drug of treatment?
D. Aspirin "
$kawasaki case
#

163-The best treatment of Kawasaki disease?


Aspirin and IVIG"$ .
#

164-kawasaki best treatment= a. aspirin b. IVIG


Keyword; Aspirin for fever, continued for 2 months. High dose IVIG is ultimate treatment

165-Child with Conjunctivitis Rash cervical lymph nodes fever what to give as a treatment?
a-Aspirin "
$
#
b-NSAID
c- No IVIG in the choices

166-poor prognosis in treating Kawasaki with IVIg? High CRP

167-A patient with kawasaki features,what is the best indicator as poor response to IVIG?
A. Neutropenia
B. High CRP "
$
#
C. Albumin
23
168-A child is complaining of 5 days of fever , bilateral non-purulent conjunctivitis , rashes in palms and
soles ( case of kawasaki ) =
C reactive protien A. Anemia "$
#

B. Neutropenia

169-Kawasaki on IVIG how to know if bad prognosis : C reactive protien


$
#
"
169-Kawasaki on IVIG how to know if bad prognosis : C reactive protien
$ A. Anemia
#
"
B. Neutropenia

170-Kawasaki case asking about which of the following is one of the criteria:
A. anterior uveitis
B. myocarditis
C. injection conjunctivitis with no exudate"
$
#

171-Fever 5 days, conjunctivitis, lymphadenopathy, high ESR and CRP. Dx?


A-Rubella
B- Kawasaki"
$
#

24
172-child with fever and then rash and peeling on hands and edema with peeling lips = kawasaki"
$
#

173-Fever 5 days, conjunctivitis, lymphadenopathy, high ESR and CRP= Kawasaki"


$
#

174-Confirmation of Kawasaki? Clinical‫"ﻣﺎﯾﺣﺗﺎﺟﻔﺣوﺻﺎﺗﻛﻠﮭﺎﻟﺳﺎﻧﺎﺣﻣروﻋﯾوﻧﺣﻣراءوﺧﻠﺻﻧﺎ‬


$
#
herpise
175-Child with vesicle at lip and gum and proximal tongue and hard palate. = gingivostomatitis"
$ ‫ﻣ ﻧ ﺎ ﺳ ﻣ ﮭ ﺎ‬
#

176-4yrs old fully vaccinated child, came to the ER with on day history of fever and sore throat which
started on the same day. What is dx?
A. scarlet fever "
$
#
B. Kawasaki
C. Measles

177-Pt with pharyngitis for 2 days , what’s the possible complication : sclar fever infectious Pharyngitis/

178-peritonsillar abscess = Quinzi"


$ ‫رﺑط = ﺑر‬peri = ‫ = ﺑرواﻟدﯾﻛﻘﺑﻼﻻﺧﺗﺑﺎر‬Quiz
#
Hot potato voice,what is the condition? Peritonsillar abscess

25
Complications :
immune-mediated complications =
• D
C
B
A
@scarlet fever@D
C
B
A
• Iacute
H
G
F
E rheumatic feverE I
H
G
F
• C
B
A
@
Dpost-streptococcal GN@ D
C
B
A
• H
G
F
E
Ireactive I ,
arthritisE
H
G
F

• SCARLET FEVER
179-acute onset of fever, sore throat, strawberry tongue 24-48 h after ((pharyngitis))) , rash begins in the
groin, axillae, neck, antecubital fossa; Pastia’s lines + may be accentuated in : flexural areas

180-A 12-year-old girl presents to her pediatrician for a sore throat. Her symptoms began approximately 1
week ago after she attended a sleepover.
Since then she experienced a sore throat and noted a temperature of 101°F (38.3°C). She denies a cough
but has noticed increasing fatigue and difficulty swallowing due to pain. On physical exam, she
hasanteriorcervical lymphadenopathyandpatchy tonsillar exudates and swelling. What complication could
he develop?
-Scarlet fever"
$
#

26
Scarlet fever is a disease which can occur as a result of a group A streptococcus (group A strep) infection,
also known as Streptococcus pyogenes .
‫اوﻟﻧرﻛزاﻧﮭﻘﺎل‬patchy tonsillar exudate‫ﻣﻧﺎﺣدﻣﺿﺎﻋﻔﺎت‬،‫ﯾﻌﻧﯾﺎﻟﺗﮭﺎﺑﺑﺎﻟﺣﻠﻘﺑﻛﺗﯾرﯾوھذاﻏﺎﻟﺑﺎﯾﻛوﻧﺑﻛﺗﯾرﯾﺎﺳﺗرﯾﺑﺗواﻛوﻛﺎس‬group A
strep‫اﻧﮭﯾﺣﺻل‬Scarlet fever i ((‫اﻟﻛﻠﻣﺔھذھﺗﻌﻧﯾﺎﻧﺎﻟﺷﺧﺻﯾﺻﺎﺑب )) ﺣرارة‬
‫ھذھﺎھﻣﻛﻠﻣﺔوﻣﻧﺎﺳﻣﮭﺎﺛﺎﻧﯾﺎﻛﻠﻣﺔ‬Scarlet‫ﯾﻌﻧﻲ‬Rash‫وﻛﻣﺎﻧﺗﺷﻣﻼﻧﺗﻔﺎﺧﻔﯾﺎﻟﻐدداﻟﻠﻣﻔﺎوﯾﺔ‬lymphoadenopathy .
‫ﻓﺎذااﺷوﻓﻧﺎواﺣدﻋﻧدھﺎﻟﺗﮭﺎﺑﺻدﯾدﯾﺑﺎﻟﺣﻠﻘوﺟﺎءﺑﻌدھﺎﺣرارةﻣﻌﺎﻧﺗﻔﺎاااﺧﺑﺎﻟﻐدةوﻛﻣﺎن‬rash‫ﻧﻔﻛراﻧﮭذاﯾﺳﻣﯨب‬scler fever . ‫ﺑﺳﺑﺑﺎﻟﮭﺟوﻣﺎﻟﺑﻛﺗﯾرﯾﻠﻠﺣﻠق‬

181-pediatric patient with fever, on examination, there is a white membrane covering the tonsils, the most
likely complication to be happened is :
A. Scarlet fever ."
$
#
B. Pharyngitis
C.Glomerulonephritis

182-Complications of tonsillitis =
A. Scarlet fever"$
#
B. Pharyngitis .
C. Glomerulonephritis

Scarlet is a condition following GABH pharyngitis or tonsillitis (exudative tonsillitis) it’s characterized by
rash (sand paper rash) and strawberry tongue
treated by abx usually one of the penicillin family

Diphtheria > the keyword is pseudomembranous tonsillitis > never complicated by scarlet or GN, it might
complicated to pneumonia

By mohtadi

183-14 yrs had fever, pharyngeal exudate, enlarged LN, most common complication = Scarlet fever

27
184-Case of infant have cough and low grade fever ,rash ,runny nose:
A. RSV" $ Respiratory syncytial virus
# bronchiolitis caused by rsv ttt ivf o2 next intubation
B. rubella Cough + Running nose + Fever

185-Child with fever and vomiting and rash on 2nd day rash become over All body
A. Meningococcemia .if with neuroligical symptome
B. Rocky mountain fever." $
#
+‫ ﻗﻲء‬+ ‫= ﺣرارة‬،‫رﻗﯾﺎ‬rash
Key word alllll body

its B if:
he mentioned hx of tick bite
or
there is hx of travel to endemic area as ( america )
or
if he said unexplained fever then yoy should consider rockey mountain fever

if no any of this then you should think of other causes, but btw if they answer that it maybe because the hx at
exam was clear its not meningococemia and other causes

186-Prophylaxis for contact with pertussis :Whooping cough = haking cough


The macrolide antibiotic = erythro*mycin*, clarithro*mycin*, and azithro*mycin*
Pertussis
Prevention is = macrloide is true

= ‫= ﺟدااﻣﻌدﯾﯾﻌﻣل‬187-whooping cagh
= ‫رﺑط‬per = ‫ = ﺑرواﻟدﯾك = ﺣﺗﯨﺗﺷوﻓﮭﺑﻌﯾﺎﻟﻛﺎﻟﻧﺗﯾﺟﺔ‬My son = mycin
Whooping cough = haking cough
188-How many years the pertussis vaccine last? 10 years
‫ﻋددﺣروف‬pertussis = 9

189-1-year-old, never vaccinated, presented with "hacking" cough and inspiratory something, What's the
organism? Pertussis."
$
#
Hemophilus influenza b

190-Pertussis with severe vomiting, most complications ?


Pneumonia ( not Pneumothoroax)
Note :Aspirations

191-4 months old with proven pertussis infection on macrolide. His 3 and 5 years old siblings are vaccinated
up to date. What is the proper action to prevent the siblings from getting the infection?
A. prophylactic macrolide" $.
#
B. booster vaccination against pertussis
C. observe them for the possibility of developing the infection.

192- 3month present with (paroxysmal cough with deep inspiration between
the cough, conjunctivitis, diarrhea, he is up to date with his vaccination.. What is the causative organisms ?
$A.
#
" Chlamydia B. Adenovirus
C. Pertussis if not vaccinated

193-Pertussis case “whooping cough”Ask about diagnosis and Investigation:


A. Nasopharyngeal swab"$
#
B. Blood culture
‫ﺣرف‬OO‫ﻣﻧﻛﻠﻣﺔ‬whooping‫ﻛﺎﻧﮭﺎ‬nose =OO.. ‫ﻓﺗﺣﺗﯾﻧﺎﻻﻧف‬
28
194-Patient 50 yrs. Old. Came for. Routine check. Up visit, by CXR you. Find.Solitary Cheast nodule.
About 2*3 patient is asymptomatic ,what is most useful thing to ask =
-Ask about old CXR" $ .
#

195-diffuse ST elevation (ECG pic) ttt? aspirin ""



$$
## Pericarditis

196-Child with Sx of varicella .Has immunodeficient brother .Action with the immunodeficient child?
A. Avoid skin contactB. Immunoglobulins"$C.
# antiviral meds

• Bacterial infection:
197-Meningitis with 1 month or less choice (ampicillin + gentamicin).
P*I*cillin = 1 month
Less than month=ampicillin and gentamicin
One month -23 month =vancomycin+cefotaxime
>24 month-50 years= vancomycine+vefotaxime+steroid
Profelax in child=rifambicine/in adult cebroflexin which is contraindicated in pregnancy

198-Meningitis with more than 1 month age choice


) ceftriaxone and vancomycin.(
‫ﺳﻔﻧوﻓﻣﺗوا = اﻛﺛرﻣﻧﺷﮭر‬

199-Neonatal lumbar puncture + diplococci Management =.


ampicillin + gentamicin"
$
#
‫اﻗﻠﻣﻧﺷﮭر‬

200- 3 days old , csf culture showed gram + bacilli catalase + beta hemolytic , how to treat?
ampicillin"#
$ "
#$
‫اﻗﻠﻣﻧﺷﮭر‬

201-15months with meningitis, Gram stain G+ double coccus tt=


A. vancomycin and gentamicin
B. ceftriaxone and vancomycin"
$
#
C. ampicillin
‫ﻋﻣرھﺎﻛﺛرﻣﻧﺷﮭر = ﻓﻣﺗواوﺳﻔن‬

202- 3day neonate with B hemolytic and catalse +ve what antibiotic give :
I )ampicillin ""

$$
##
2)gentamicin
3 (ceftriaxone

203-Child 15 month with meningitis lp show gram + cocci mange?


A. Vanco+ceftr"$
#
B. ampicillin +gentamicin

204- 15m old baby with sign of meningitis on csf analysis you found gram positive diplococcai treatment ?
A.ampocilib B.voncomycin
C. ampicilibe with guntamycin
D. vancomycin with ceftraixon"$
#

205-8 years old child came with fever ,neck stiffness ,irritability, normal glucose normal protein ,increases
WBC what's abx used ?

29
A- Ampicillin .
B- Ceftriaxone .
C- Vancomycin .
D- Ceftriaxone and vancomycin"
$ .
#

206- 7 Y with meningitis , Gram stain G postive diplococci tt= :


ceftriaxone and vancomyci . ‫ﻻزﻣﺎﻻﺛﻧﯾﻧﻣﻌﺑﻌض‬

207- Child with pain that last for 10-15 mints (intermitted pain) crying , postive kering sign ? What is the
best treatment : Ceftrixon + Vanco 🍒 pKernig's sign = meningitisl
o
n
m
l p
o
n
m
7y/o with meningeal irritation, headache, and fever .
CSF:
Bacteria:low gelcos and hight protein high neutrophils
Viral: high protein and normal gelcos
TB:high protein.low gelecose but hight lymphocyte

208- CSF (normal protein and normal glucose and lymphocytosis) .


What you will give the child ?
A.Ceftriaxone and vancomycin and steroids
B.ceftriaxone and steroid
C. antiviral ""

$$
## Bez( normal glucose )

209- Pt 7 yrs old with nausea and vomiting dehydrated comatose acidotic CSF : high protein , normal
glucose
viral meningitis"
$
#

210- 5y/o with fever, lethargy, positive Brudzinski sign. CSF showed lower limit of glucose with high
protein. Gram stain revealed gram positive cocci in chains. Your management ?
A. Ceftriaxone, vancomycin, and steroid" $ *
#
B.ceftriaxone and steroid
C.antiviral

211- 7y/o with meningeal signs, headache, and fever. He and his family came from Africa recently. He also
has sore throat and lymphadenopathy. CSF (normal protein and normal glucose and lymphocytosis). What
you will give the child ?
A.Coronavirus
B.CMV
C. EBV" $
#
Hemorrgaic fever comes with Ebola, cerebral malaris. So alwyes put ebola at the last choice unless he
mentioned hemorrgaic and fever then put ebola at the top

212- Fever for 6 days and tender splenomegaly, which culture is most importantly needed?
-urine and stool culture
-repeated blood cultures" $
#
- bone marrow smear culture
. ‫ﻧﻔﻛرﻓﯾﺎﻟﺗﺎﯾﻔوﯾد‬،‫ اﯾﺎﻣوﯾﻛوﻧﻔﯾﮭﺳﺑﻠﯾﻧوﻣﯾﺟﺎﻟﻲ‬٦ ‫ﻟﻣﺎاﻟﺣرارةﺗزﯾدﻋن‬
213- Baby mele fever wet his diapers what is suggests uti in that baby wetting dippers boy == fever"
$
#
If sex is boy then US or structural abnormality is the choice if girl then fever
‫اﻟﺳؤواﻟﻘﺻدھﺎﯾﺷﺎﻟﯾﯾﺧﻠﯾﻧﯾﺄﺷﻛﺎﻛﺛراﻧﺳﺑﺑﻛﺛرةاﻟﺗﺑوﻟﮭوﺑﺳﺑب‬UTI‫ﻻﻧﻣﻧﺎﻋراض‬،‫وھووﺟوداﻟﺣرارة‬urinary tract
infection. ‫ﺧﺎﺻﺔﻓﯾﺎﻻطﻔﺎﻟﮭواﻟﺣراااارةﻣﻌﮭﺎﻛﺛرةاﻟﺗﺑول‬

214- 15old fever, abdominal pain, splenomegaly = multiple blood cultur

215- Girl 7 years old with suprapublc pain No rebound no guarding. Tx ?


30
A-observ B-admits iv abx
C- discharge with oral Abx"
$=
# diagnosis interctetial cystitis ‫ﻣﺿﺎد ﺣﯾوي وﺧﻼص‬
‫ال‬UTI‫ﻣﺎﯾﺣﺗﺎﺟﺗﻧوﯾم‬

216- 7year old child, presented to the emergency by his partners with 2 days history of fever and vomiting,
child is comatose dehydrated with acidosis. CSF report: Cells 20 (above normal) , Protein 0.45 (above
normal) ,Glucose (Normal)
A. Tubercular meningitis .
B. Salicylate toxicity.
C. Diabetic coma .
D. Viral meningitis"$ . high protein and normal gelcose viral ‫واﻧت ﻣﻐﻣض‬
#

217- child with chill ,fever ,irritability , on exam there was neck rigidity and positive Kernig’s,CSF showed
clear color, lymph and neutrophils high, protin and glucose normal, dx?
A. pneumococcal meningitis
B. Meningococcal meningitis‫ﻻﻧﺎﻟﺑروﺗﯾﻧﻧورﻣﺎل‬
C. aseptic meningitis"$ Viral meningitis
#
D. pyogenic meningitis

218- Girl ((confused )) with fever + sign of meningitis = encephalitiisq


u ‫ﻟﻣن ﯾﺟﻲ‬pt with sign of mengities
t
s
r
and alter conscious level

219- child meningitis, what is the complication ?


A. blindness
B. hearing loss"
$
#

220- Chime with meningitis came with his parents and has papilldema , parents are afarid of ?
A-Hearing loss "$
#
B-Vision loss
If he said only papilledema it means vision loss. So the answer he is hearing loss not vision loss

221- child with (Irritability) headache ,nausea, lethargy and rash all over the body what is dx :
meningococcemia
$ B. kawasaki
#
"

222- 3mo old boy with pic of bacterial meningitis،most common pathogen?
A. Moraxella catarrhalis .
B. Streptococcus pneumonia"$ so BGS, pneumonia most common in infant
#
C. Streptococcus pyogen

223- pediatric patient have meningitis, with close contact to his brother recently, Asking for what to give to
his brother: A. Rifampicin "
$
#
B. IVIG
So in childe we use rifampicin but in adult we use cebrophlexine

224- Most common cause of ear infection or otitis media in children?


A. Bacteria "
$
#
B. Virus
C. Fungal
31
224- Child came from africa. complaining of weakness, he couldn’t move his head and legs especially when
he is prone. What is the dx?
A. Polio "
$
#
B. Cmv
‫ال‬polio‫ﯾﻌﻣﻠﺷﻠﻼﻟﻌﺿﻼت‬
Key words: back,neck,arms and legs pain and stifness

225- At school age what we tend to prevent?


A. Japanes encephalitis
B. Hemophilus influenza encephalitis"$ most common school age for this reason any child joined
#
school give him this vacine
‫اﻟﻣدرﺳﺔﻛﻠﮭﺎاﻧﻔﻠوﻧزا‬
C. Herpes encephalitis

226- Children with 2 days history of ear pain, exam reveals perforated ear drum with fluid passing through
it =
A) Chronic otitis mediaB)Acute suppurative otitis media"$C)
# Otitis externa

227- Most common virus cause of acute otitis media in pedia =Rhinovirus or RSV is the first most common
= ‫رﺑط‬rhino = ‫ = ﯾﻌﻧﯾﺎﻧف‬ottits‫اذﻧﺎﻻﺛﻧﯾﻧﻣرﺗﺑطﯾﻧﺑﺑﻌض‬
Virus rhino bacteria strepto pneumonia

228- Child hasfever with perforated tympanic member and pus in the external canal = Acute OM ( Not
Otitis media with effusion )
Ttt amoxacilne high dose. Pt not ttt with antb so pt came with perforated

229- child came with limping and non-weight bearing, not allowing anyone to touch his leg, most common
organism is: staph. Aureus " $+(septic arthritis)
#
، ( ‫رﺑط = ﺳﺗﺎف = )ﻋﻠﯨﺎﻟﺣﺎﻓﺔﻣﺎﯾﻘدراﺣدﻣﺟردﺑﺳﯾﻠﻣﺳﮫ‬aureus. ‫ﻻﺣدﯾﻠﻣﺳﮫ‬،‫= اوھﺑﺳﯾﺄﻟم‬
‫اﻟﻣﻔروض اﻟﺳوال أوﺿﺢ ﻣن ﻛدا ﯾﻘوﻟك ان ﻓﺑﮫ اﻟم ﻓﻲ ال‬knee joint and the child limitation movment and joint hot
Which means arthrities
230- child with fever and left knee pain and swelling. Most important single investigation?
A. Blood culture .
B. FBC .
C. Joint aspirate ."# $ "
#$
D. Xray

231- Conductive hearing loss could be because of recurrent OM.

233- Impetigo causative organism honey crust lesion:

A-staph aureus."
$
#
B-GBS
C-Pdeudomonas

234- Pediatric has unilateral hip joint pain Flexing Refuse to be touched Mostly causative organism ?
A. Staph aurus"#
$ "
#$
B.Group b strep vvvC.H influnza

235- child with limping for 2 day and abnormality in hip what's most likely organism=
A-staph "
$.
#
B. GBS
C. Hib
32
=‫ﺳﺗﺎف = ﺗﻔﺎف‬hip (:‫= ھب = ﺣﺳد‬

236- Child with pneumonia, indication of hospitalization ?


A. Vital signs( hypotension and tachycardia)
B. not able to take orally"
$
#

237- 4years old with pneumonia, with vomiting for any oral food .. vitaly stable; reason for admission:
A. pneumonia
B. unable to tolerate orally"
$
#

238- headache, stiff neck, and vomiting, coughing and breathing with breathing difficulty , causative
organism ?
strep. Pneumonia
Pediatric Pneumonia=
A. Iv antibiotic ✅✅
B. B_ oral Abx
‫اﻟﺗﮭﺎﺑرؤﯾﺷﻲءﺧطﯾرﻻزﻣﻌﻠﯨﺎﻟورﯾد‬

238- 13months old girl present with fever 38 , bilateral lung infiltrate , she looks mildly ill , what is the
likely organism==
A. Moraxella catarrhalis
B. Strep pneumoniae"$
#
C. Hib influenza

239- Baby with tachypnea, cough, hemoptysis and bilateral lung infiltrates. What is the treatment?
A. Steroid .
B. Antibiotic."
$
#
C. Surgery.
D. Betablocker

240- child with flu like symptoms+ fever + has middle lobe crackles+ stony dullness, chest x.ray will
show = pleural effusion bez dulness not pnimothorax
Key word stony dullness it means pleural effusion. But pneumothorax It mean hyper resonant

241-Staccato cough : chlamydia pneumonia


. ‫ﺳﺗﺎﻛوا = ﺳﻛوت = ﻛﻼﻣﻛﺛﯾر‬

242- Children with cough, fatigue, 2 time bloody vomiting, low grade fever, with dullness in percussion dx?
A. parapneumonic effusion"$
#
B. pleural effusion
C. TB
Key word dullness and bloody vomiting but its better in the question if he said hemposis or coughing blood

243- Absolutely contraindication in penicillin allergy?


Pipracillin/tazobactam

244- Child with poor feeding since 2 days have oral thrush and dipper dermatitis what you will give
A. Topical antifungal .
B. Oral antifungal .
C. Topical and oral anti fungal"
$
#
D. Systemic antifungal

245- pic of Cystourethrogram of a pediatric with recurrent UTI =

33
VesicoUrethral reflux is a condition in which urine flows backward from the . bladder to one or both ureters
and sometimes to the kidneys

246- Patient presented with typical picture of malaria infection, blood smear showed no
parasite, what’s your next step?
A. Repeat thin blood smear.
B. Repeat thick blood smear
Repeat every 8 hours for two days"$
#
‫ ﺳﺎﻋﺎت‬٨ ‫ﻣﻼرﯾﺎ = ﻣﻼﯾﺔ = ﻧﻧﺎﻣﻌﻠﯾﮭﺎ‬

247- Tonslitis = amoxacillin / clavi.

248- Neonate with sign of sepsis what is empirical antibiotic ?. Ampicillin

Cystitis in child ?
249- Q about treatment of uncomplicated cytits
A.iv ceftrixone
B.im ceftrixone
C. oral amoxicilln ."#
$ "
#$

250-Pediatric prophylaxis formaningitis? rifampicineq


u
t
s
r

251- Young girl with diarrhea came with left knee swelling, right elbow, left Achilles tendon. Stool analysis
shows +ve clostridium toxins. What is the dx?
A. JIA
B. Reiter's syndrome
C. septic arthritis
D. Reactive arthritis"$
#
Reactive typical sites > knee and ankle
So reactive arthritis Reiter syndrome response to infection in another part of the body cross reactive

252- Baby has swollen tender joint, limited passive movement, the most important test:
Examination of synovial fluid

253- When resuscitating a child with septic shock, least evidence of benefit in treatment= steroids"#
$ "
#$
‫الخمول‬
254- Ped URTI lethargy , confuse , Tem39, rr35 , hr>100 = .Sepsis"
$
#

255- hypotension and not respond to ivf = septic shock


key words not respond to iv

256- Commonest cause of shock in children? sepsis"


$
#

257- Neonate with High fever, developed petechial rash and was hypotensive 70/55, with cold extremities
and poor feeding. What is the dx :A. septic shock" $
#
Cold extrimities septic shock its late also for hypovomic but not early septic.

258- child with leukemia , after 17 days of chemo coming with fever , neutrophil is normal, tx ?
blood culture , urine culture , broad spectrum iv abx

259- Child with fever, vomiting and diarrhea. ABG: Normal pco2 + Normal bicarb Base access -4
“ normal range from -2 to 2 ”
Ph 7.3
34
Ph 7.3

260- What type of shock does he have ?


early compensated shock "
$ Not late
#

261- Pedia pt under went tonsillectomy, the father notes that surgery take more time than usual but pt
recurved well, thre was bleeding during surgery they ask what you tell the father? Explain to the father what
happenedqu
t
s
r

262- Child with flu symptoms on examination he has membranous exodus in tonsils, causative organism
A. streptococcal "
$.
#
B. EBV‫ﻓﺎﯾرس‬

263- child present to the clinic with pain in micturition and lower abdominal pain
all symptoms of UTI and his parents observe in his urine foul smell, which organis can cause this condition?
A. E coli gram negative."$
#
B. E coli rod in shape.
c. Proteus gram negative bacill
،‫ يشعر املريض بعدم القدرة أو صعوبة في تفريغ األمعاء عند التغوط‬.‫ هو شعور بالتغوط الغير مكتمل‬، ‫زحير مستقيمي‬
264- 10 Y boy with hx of 2wk of bloody diarrhea and abdominal pain, tenesmus: Amebiasis
‫ﻛرﯾﻣﻣﯾﺑوا = ﯾﺧﻔﻔﺎﻟﺣروﻗﺎﻟﯾﺑﯾﻧزﻟدم‬

265- Child c/o fever, bloody stool, and tenesmus, abdominal exam showed abdominal distention, Dx?
.Amebiasis."
$
#
Key words tenesmus

266- child has(( bloody diarrhea)) and oliguria, vomiting, nausea, abdominal pain, fever for 7 days before
that family think this from restaurant What's ,treat =
A.- antibiotics"
$
#
B.- supportive
C.- steroid

267- 7 years with sore throat + difficulty swallowing + painful cervical lymph nodes. the organism =
streptococcus pyogenes

268- Child with group A strep pharyngitis. What will you do with his brother?
A. Observation " $
#
B. Throat culture
C. Throat swab for rapid antigen test
D. Antibiotics
Baby with mass in umbilical and developed veslculopustular rash grape like organism = . Group A strep"
$
#
‫ﺳﺗرﯾﺑﺗوا = ﺳﺗر = اﺳﺗر‬umbilical
• Neuropedia
269- child took hyoscine butylbromide and metoclopramide for gastroenteritis and fulud the he develop
jerky movement what to give ? domperidon
= ‫رﺑط = ﺟﯾرك‬jerk = ‫ = ﺟﺎﻛﯾت‬domper‫= دوﻣﺑﯾر = ﺑردﺗﺑرد‬

270- Fm or pediatric had gastoentritis she took metoclopramide that leads to involuntary
movment facial grimace and tongue protroduing what to give:
&
A. Diphenhydramine $
%
B. Epinphrin
C. Tizandine
Answer is A
35
271- child starting (brief sizer) (less than (30 seconds) , EEG (generalized 3-Hz spike-and-wave activity.)
Treatment = ethosuximide ( Absence seizure (

272- Child fall from his bed complain of headache and 2time vomit everything normal next= close
observation

273- An 8 year episodes where he ((blinks multiple times)) and becomes okay after that.. he is conscious and
responsive during those episodes .
&
A- TICS$
%
B- TOURETTE SYNDEROM
C- BLINKING DISORDER

274- Addictive found in (deep coma)


and cyanosed what do to?
a-Mechanical ventilation"$
#
b-History details from family

275- Fracture of the left stylomastoid foramen during delivery of a baby: ( can not open eye )

Loss of anterior 2/3 sensation of tongue

276- child unable to feed herself with a spoon , hx of head trauma 10 days lesions? cerebellum

278- pt BMI 18 but think of oneself as obese thus dieting, Dx ?


A. bulimia‫ﺗﺎﻛﻠﻛﺛﯾرﺑﻌدﯾﻧﺗﺳﺗﻔرﻏﺎﻻﻛل‬
B. anorexia"
$
#

279- during delivery absent (moroʼs reflex on right side= Erbʼs palsy" $
#
CLINICAL PRESNETATION:
Waiter tip position. Shoudluer addcut internal rotate, wrist reflex. Asymmetrical moros reflex

280- absent hand motor reflex === Erbs palsy

Earb’s palsy, happened to macrosomia baby Wight more than 4500

281- Child has pneumonia+ fever then developed seizures. = diazepam

282- Child with febrile illness and seizure


Rectal diazepam"#
$ "
#$

283- Continuing >35 minutes Seizure epilepticus Given lorazepam iv what's next= Iv phenytoin

284- Child k/c of grand mal seizure on Depakine comes with breakthrough seizure
what to give initially after intubation on ER presentation: Diazepam

285- Pt status epilepticus for 5 min, with iv access, what is the first line:. lorazepam

286- kid with seizure for more than 5 mins, iv line secured = IV lorazepam

287- Cerebral palsy in kernicterus?


(Athetoid) cerebral palsy or (dyskinetic) cerebral palsy"
$.
#

288- A case of icterus > Kernicterus


-1ABO incompatibility" $
#
36
-2RH incompatibility"
$
#

289-Fist hand +feet crossed seen in:


cerebral palsy"
$
#

290- 6w or month old with unilateral absence of red reflex what to do ( next)
A-Mri brain and optic .
B-Funduscopy" $
#

monthes
291- best time to do red reflex examinations =at birth and age of 6 weeks
Reflex = 6 monthes

292- 15months old baby presented to the clinic with developmental delay, on examination patient was
having spasticity, crossed leg, lower limb were involved more than the upper limb but both were affected,
what’s the type of cerebral palsy he has :
A. Quadriplegia.✅
Diplegia upper ,lower more in lower
Quadriplegia upper ,lower more in upper
Cross legs then its quadriplegia
If the knee inward meet then its diplegia

293- girl with bruises and fecal incontinence after being fully toilet trained: dx sexual abuse"
$
#
itching bleeding offensive smell= forign body
laceration or urine stool incontenice = sexual abuse

294- Parents came with there child complaining of that there child( always blinking on rest ) and activity on
examination there is no pain and tearing what is the most likely diagnosis‫? ﯾﻌﻧﯾﯾﻐﻠﻘﻌﯾوﻧﮭﺑﻘوة‬
Tics disease"
$
#
= ‫رﺑط‬tics‫ = ﻣن‬tight. ‫ﯾﻐﻠﻘﻌﯾﻧﮭﺑﻘوة‬،‫ﺑﻘوة‬
Shaken baby syndrome:
Retinal hemmorage
Brain sweeling
Subdural hematoma
295- Child with rapid eye, he is active with parents during the attack =Tic disorder

296- Child with repetitive eye movements. Otherwise normal no pain nothing. On exam no redness or
tenderness or edema. Dx ?
Tics syndrome
= ‫رﺑط‬Tics = ‫ = ﺗﺎﻛﺳﻲ‬repetitive eye movementsƒ
yz
~•
ˆ



„ x .‫ﺟﺎﻟﺳﯾﻧظرﺑﻛﻠﻣﻛﺎﻧﯾدورﻋﻠﯨﺗﺎﻛﺳﻲ‬
}
|
{


297- 5 years old presents with inability to stand or sit unsupported and clumpsy gait and resistance to
Acute cerebellar ataxia"
neck flexion after chicken pox infection weeks ago:dx: Meningoencephalitis $
#
‫رﻛزﺑﺎﻟﺳؤواﻟﯾﻘوﻟﻛﺑﻌداﺻﺎﺑﺔﺑﻌدوﯨﺣﺻﻠﻠﮭﺧﻠﻠﻔﯾﺎﻟﺗوازﻧﻠﻠﻣﺧﯾﺧﺎﻟﯾﮭو‬celebrum‫وھوﺑﺳﺑﺑﺎﺻﺎﺑﺗﮭﺑﻌدوى‬
Acute cerebellar ataxia of childhood is a childhood condition characterized by an unsteady gait

298- Case of abuse, subdural hematoma and retinal hemorrhage , the childʼs mother is mentally retarded
but father is normal = shaken baby syndrome‫"ﻻﻧﻘﺎﻟﻠﻛﻧزﯾﻔﺑﺎﻟﻣﺧواﻟﻌﯾن = ﯾﻌﻧﯾواﺿﺣﺎﻧﮭﮭزةﻗوﯾﺔ‬
$
#

299- fetus delivered by Vacuum instrument, has swelling that doesn’t cross the sutures? Cephalohematoma

300- Child Head trauma subarachnoid, hge, periorbital edema, bruises and LOC Father said he found her
like this, what to do? Call child protection

37
301- Can’t close his eye inone sidewhat nerve affected? -Facial Nerve‫"ﻣﻧﺎﺳﻣﮭﺎﻋﺻﺑﺎﻟوﺟﮫ = ﻋﯾن‬
$
#

302- Duchenne muscular dystrophy sign?


A- Gowers maneuver✔✅
B- Hypotrophy of the calf
.‫ﺟوارس =ﻗوم= ﺣﺎوﻟﺗﻘوم‬
Bottom bowers maneurver an early sighn of Duchenne mascular dystpohy, consiste of boy pushing his hand
against his knee then thigh to reach a standing position.
303- Child afraid of going to school, how can the mother deal with the case =
Talk to him about how his favorite super hero would deal with the situation.positive reward

304- 1x2 cm on the forearm since birth, no symptoms =follow up

305- 20 years old with a skin tag lesion 1-2 cm in forarm. No change since birth no further description or
color of the lesion mangment:
&
A- Reassure$
%
B- Laser
C- biopsy

306- What improved to decrease the premature baby mortality rate? hypothermia
PremTure = hypoth
Hypothermia < 36 is a major factor in morbidity and mortality in premature infant and do high risk
intraventricular damage and coz hemorrage

• Bronchiolitis.
307- Case of bronchiolitis (severe) with chest recessions management?
Admit for fluids hydration and oxygen." $
#

308- Portia pt have tachypnea, runny nose, cough,slightly elevated fever, audible wheezing sound whats is
tho definitive diagnosis: = Nasopharyngeal swabs (case of Bronchiolitis caused by RSV)" $
#

309- bronchiolitis, developed several episodes of apnea = .


a. Supportive
b. Ventilatory management" $
#
c. IV methylprednisolone

310- two cases about bronchiolitis: cause and treatment= RSV. Ttt: supportive rehydration"
$
#

311- pt have arthralgia after viral infection" watery diarrhoea " what is the type of arthritis?! Reactive
Chronic productive cough , SOB wHt dx? Bronchiolitis = RSV = supportive

312- Bronchiolitis:RSVrespiratory syncytial virus.

313- Child with Small VSD, asymptotic= observation and watchful waiting"
$.
#

314- Pediatric with intact radial and reduced femoral, with hx of htn: Coarctation of the aorta

315- best diagnostic for CoA - Coarctation of the aorta for neonate ? Echo
CoA = Cho = E*cho*

316- Child need non invasive M.V =icuErpeditric


room

38
317- Child with fever and bilateral conjunctivitis and abdominal pain w
on x-ray follow up you found bilateral lung infiltration, most common pathogen?
A. Moraxella catarrhalis
B. Streptococcus pyogen
C. Adenovirus" $
#
*bialatral* lung infiltration = Viral

• Pediatric surgery
318- 2year old boy with pain over anterior tibial tubercle, diagnosis?
A- Osgood Schlatter Syndrome

319- RTA and you find aortic thoracic injuries and splenic abrasion with hypotension what u do?
A. Thoracic surgery

320- positive rebound tenderness in Macc Barney point (case of appendicitis) the pathophysiology =
peripheral vasoconstriction

321- 8old with RLQ pain and rebound tenderness what's confirmatory test?
A. US abdomen" $
#
child having sudden pain at thigh pic of spiral fracture with labs of (PTH, Ca) high= primary
hyperparathyroid"$
#

323- 9y old child , PTH high , ca high , came with bone pain tt ?
B.rehydration + diuretics + bisphosphonate" $
#
..‫ﻣدراﺗﻌﺷﺎﻧﻧطﻠﻌﺎﻟﻛﺎﻟﺳﯾوم‬
324- Bowing legs, frontal bossing , manag= Vit D3

325- child with rash appeared as vesicles. . He has immunodeficient brother. Family has concern about their
immunodeficient child =
A.give oral antiviral
B.don’t do anything because this disease is self-limited
C. give IVIG."
$
#

326- boy came with deep stabbed wound in the anterior right thigh 10 cm in depth. What is your next step ?
A.tourniquet
B.call vascular surgeon
C. direct pressure apply on the wound"
$ *
#
D. directly clamp the femoral artery

327- Child abdominal trauma, investigators show splenic lacerations 2cm w peri-spleen
fluid most appropriate management A. Non OPrative‫"ﯾﻌﻧﯾﻣﺎﯾﺣﺗﺎﺟﻌﻣﻠﯾﺔ‬
$
#

328- child fall dawn on his hand ( radial&ulna) fracture,1cm open wound
A.Closed reduction with cast above elbow.
B.Closed reduction with cast bellow elbow
C. Innernal fix with cast till elbow.
D. surgical debridement& fixation" $
#
‫ﻧﺧﺗﺎردي‬،‫ﺑﺳﻠوﻣﺎﻓﯾﮭﻛﺳراوﻛﺳرﺑﺳﯾطﻣﺎﺗﺣرﻛﺎﻟﻌظﻣﻣﻧﻣﻛﺎﻧﺔ‬،‫ھوطﺑﻌﺎﻋﻠﯨﺣﺳﺑﺎﻻﺷﻌﺔ‬

329- Child with humural & ulnar & and un able to move extensors muscle of forearme and hand ?
median nerve in cubtal fossa
330- Child with vascular malformation of lower limb , when to interfere :
39
A- pain✅✅
B- B- Claudication

331- Newborn circumcision , chordee and hooded foreskin + hypospadias how to manage ?
A.open circumcision
B. plastibell circumcision "
$
#
C.circumcision with
D.referral to pedia surgeon
‫ﺑﺳﺎﻟﺻﺣﺑﻲ‬،‫ﺳﺎﺑﻘﺎﺣﻠﯾﻧﺎھﺎﻋﻠﯨﺎﺳﺎﺳدي‬

332- infant with hypospadias and for circumcision, what’s the procedure?
The surgeon will use a small piece of foreskin to create a tube that
increases the length of the urethra"
$
#

333- Bleeding Circumcision which factor ? 8


= ‫رﺑط‬Circumcise= Circumcision8 ‫= ﻋددﺣروﻓﮭﺎ‬

334- Neonate+ prolonged bleeding after circumcision, aptt high, pt, bleeding time and platelet are narmal,
condition is most likely due to deficiency in
Vii
Bleeding after circumcision factor 8
Bleeding after umbilical stump> factor 13
Bleeding after home delivery - Vit K (factor 10)
10/10 ‫ﻧﺎﺧذراﺣﺗﻧﺎﻣرةﺑﺎﻟﺑﯾت‬

335- home delivery baby with umbilical bleeding after day 5, dx:
A- vit k deficiency
B- factor X deficiency ."
$
#
)(umbilical = à fsctor) (

339- infant had bruises on his thigh after delivery at home what is the cause?
vit k def

334- boy come with gingival hyperatrophy with enlarge and bluish purple friable gum . What’s vitamin
deficiency?
A. vitamin A
B. vitamin C"
$
#

335- Boy bleeds out after tooth extraction Then after develop bruises they mention factors Vlll = +ve , Plat=
normal, Only slight increase in pt Dx= von willebrand disease
V = III (VON)‫ﺛﻼﺛﺣروﻓﻣﻌﺛﻼﺛﺎي‬

336- Baby for circumcision u found urethra mid shaft what procedure will u do
a. gomco clamp
b. plastibell
C. inform surgeon"$
#

337-Months boy Uncircumcised with fever , labs showed UTI=


IV Ceftriaxone "
$ cut‫= ﺳﯾف‬
#
‫اﻧﺗﺑﮭوا‬I.V‫وﻟﯾس‬Orall .

339-most likely indicates he has UTI: Nitrate

40
340- 4months Child with mid-shaft hypospadias, came for circumcision. What u will do?
A- not possible since they will use it for the repair"
$.
#

341- Child circumcised has UTI treated with TMP/SMX and improved, which further should be done?
A- renal ultrasound
B- cystourethro
C- reassuring"$
#

342- presented with abdominal pain and jelly like stools) what’s the best diagnostic test?
A. Abdominal U/S .
D. Barium enema" $
#

343- jelly like stools, = the best diagnostic test = Barium enema
Intussusception, one about next step after stabilizing pt: .
barium enema / radiological reduction" $
#

-344- intussusception case, what would you tell the mother?


A- Shock is most common complication
B- Require surgery immediately
C- Recurrence common after surgery" $
#

345-Case of baby with intermitted cries that is sever to the point where he is rise his legs and screams for
hours what best nexst step= abdominal ultrasoud (intussusception )
‫ ﺳﯾﺳﯾﺑﺷن = ﺳﻛﯾن = اﻟﻣﺷدﯾد‬. ‫ﻟﻔﺗراﺗﻣﺗﻘطﻌﺔ‬،‫وﯾرﻓﻌﻘدﻣﮭﻣﻧﻘوةاﻻﻟم‬،‫طﻔﻠﯾﺻرﺧﻣﻧﺎﻻﻟم‬

346- Tt of intussusception in pediatric = hydrostatic enema =


)Dont choice reduction by ultrasound or fluroscopic(

347- Abdominal exam sausage shaped mass = Intussusception

348-intussusception case how you confirm it clinically :


A- Sausage mass"#$ "
#$
B- Red currant jelly stool

349-Intussusception case stable Best initial radiological reduction ( not I.v fluid bez he is stable)
Child cry when left her hip with mass in upper abdomen what’s Dx: intussusception

350-Intussusception child first thing to do = Iv fluid & analgesia"


$
#

351- 13month old with Abdominal tenderness, vomiting ,bloody stool,leukocytosis and US *doughnut*
shape. intussusception

352- 3year old girl with bloody diapers. She has no pain or constipation.diagnosis=
A.Meckel's Diverticulum" $
#
B. Intussusception
C. Colon

353- target sign by us= Intussusception


Case of intussuption child very dehydrated what is the next immediate action? Iv fluid

354- 11months presented with bloody smelling stool how yo confirm dx


A. Us✅$
#
"
B. Barium enema
41
355- Intersection presentation initial investigation: = ultrasound abdomin

356- case of intussuception came with clicky pain + doughnuat sign on ultrasound + bloody stool what is
most important step to manage rhis case ?
Urgent surgery
Nasogastric decomprestion
I.v fluid resuction"
$
#
Bariam enema

357-Neonate with bilios vomiting He pass meconiun (Then he pass yellow stool) = -Mid gut volvulus

357- Child with bilious vomiting and pass limited amount stool. 5 days old = Volvlus"
$
#

358- baby 3-7 days presented with ((bilious vomiting )) decrease oral intake , this happened after
introducing milk formula He passed meconium after birth and after that yellow stool:
B. milk allergy C. Mid gut volvulus"
$
#
‫ﻛﻠﻣﺔ‬bilous‫ﻣﻌﻧﺎھﺎاﻧﺳداد‬

359- Coffee bean sign and want the diagnosis? Sigmoid vulvulus

360- Thumb print sign in abdomen and eant the daignosis ? Bowel ischemia

361- Abdominal distrnsion, constipation, vomiting. No Xray in choice investigation =


A- CT
B- B- US✅

362- Primary sclerosing cholangitis What to do?Colonoscopy.

363- Child tripped on a toy and the right leg was trapped within the toy and fell on the leg child
complaining of pain , what type of fracture do you expect-:
-Spiral facture of femur
-spiral fracture of tibial"
$
#
Tibia= Toy

364- A child was brought by his parents after he refused to walk and insisted on being
carried always, the parents reported this happened after he was playing and
stepped on a toy and his leg was twisted and fell down = Toddlers =spiral fracture of distal tibia.

365- Child with X ray of distal radial and ulnar bone fractures =
Cast below elbow

367- 6y.o child with fracture of thigh and 30% angulation. TTT?
Hip spica with traction

368- Child with forarm fracture , mx ? closed reduction and cast

369- Distal radial fracture in peds patient (xray shown), partially penetrated the skin (picture).= internal
fixation with casting below elbow" $
#

42
370- Child fall from hight presented to you in ER crying, Bleeding from the ear, tympanic membrane
bulging and bleed , imaging confirmed basal skull fracture, The nerve which pass through foramen ovalea
injured whatʼs the function which will be affected ? Mandible nerve , mastication
= ‫رﺑط‬Mandible = man
foramen = men = man = (man with men)

371- child with supracondylar fracture, distal pulse not palpable, your management:
A- K-wires .
B- exploratory operation ."#
$ "
#$ closed reduction and cast
C- hand elevatio

372- xray of both bone distal forearm fracture, greenstick, your management:
closed reduction and cast
Green = cloklsed = cloth‫اﺧﺿر‬

373- A 3 y/o fell off his bunk bod and immediately cried, but ho didn't have an apparent loss of
consciousness. Brought to tho hospital and further Investigations show that there l$ no skull fracture. What
is your next action? . Watchful waiting

374- Infant with absent red reflex ( retinoblastoma most likely) What to do :
Immediate referral to ophthalmolog

375- what diagnostic for retinoblastoma ? CT‫ﻋﺷﺎﻧﻧﺷوﻓﺎﻟورﻣﺎﻧﺗﺷروﻻﻻ‬

376- Retinoblastoma on slit lamp examination = urgent referral to ophthalmologist"#


$ "
#$

• Autosomal Recessive
inheritance
= ‫اﯾﺳؤواﻟﻔﯾﮭرﻗﻣﻣﻌﻧﺎھﺎ‬R = Recessive

377- 17alpha hydroxylase deficiency= autosomal recessive

371-Baby with ambiguous genitalia what deficiency: 21 hydroxylase"


$
#
= ‫اﻣﺑﯾﺞ‬ambig‫ ﷼‬21 ‫ب‬Š‹
• Š‫= ھﻣﺑرﺟ ر‬
Ž•
•Ž
Υ
‹Œ

372- inheritance of an ambiguous genitalia= AR (21 hydroxylase deficiency)

373- Pt carry risk of 25% to have genetic dis , What is the type of genetic abnormality?
A. autosomal recessive
B. autosomal dominant

374- Parents carrier 25% chance of having affected child with "Cystic fibrosis"= autosomal recessive
=‫رﺑط‬cystic = ‫ = ﻛﯾس‬receive (: ‫اﺳﺗﻠﻣﺗﺎﻟﺑﺿﺎﻋﺔوﺣطﯾﺗﮭﺎﺑﺎﻟﻛﯾس‬

375- Child with cough,wheezing , recurrent infection, poor feeding and poor weight gain and murmur
=Cystic fibrosis

376- Congenital adrenal hyperplasia mode of inheritance ?


AR 21 hydroxylase deficiency
case of Neurofibromatosis type 1 (7 cafe au liat spots, axillary freckles), ask mode of inheritance :autosomal
dominant
. ‫ = دوﻣﯾﻧﺎت‬.‫ﻧﺷﺗرﯾﻛﺎﻓﯾﻣﻧدوﻣﯾﻧز‬Dominat
43
377- What is the type of genetics in pt presented with cafe aule spots dx neurofibromatosis? -AD"
$
#

378- Child with multiple cafe au let spot on his body, his mother mentioned that his relatives also have the
same spots= consul about NF1

379- 4year old boy brought by his mother examination reveals multiple Café au lait spots. The mother says
that “it’s a common birthmark in our family” what is your test action =
A. Send her for genetic counseling"
$
#
B. Educate her on NF type 1
C. Confirm diagnosis with genetic studies
D. Reassurance

380- case about wiskott-aldrich ( recurrent infections, eczema, 2 healthy sisters, 1 died before the age of 10
months ask about mode of inheritance? x linke
‫رﺑط‬
Died = X wiskot = ‫ = اﺳﻛت‬X reccurnt infection = ‫ = ﻛﻠﺷوﯾﻌدوﯨﻌدوى‬X

381- Case of 14m boy with a history of 4 lung infections, he has to healthy sisters. X-linked
agammaglobulinemia
Sister = X
#Wiskott–Aldrich syndrome characterized by 3 thing :
eczema + thrombocytopenia + immune deficiency or upper resp infection
#Wisket = Weak‫= ﻣﻧﺎﻋﺔﺿﻌﯾﻔﺔ‬
#Aldrich‫= اﻛزﯾﻣﯾﺎ = ھرش‬
#Wisko*tt*= thrombocytopenia

382- Child with recurrent URTIs , eczema and thrombocytopenia both brother and uncle have the same
condition? Wiskott aldrich

383- Child with tuberous sclerosis ?


A. Single gene testing
B. Multiple panel gene testing"
$
#
‫ﺳؤواﻟﻌﻠﯾﮭﺎﺧﺗﻼف‬

• Endocrine
384- recommended screening age for hemoglobin? 12 mo

385- eye pigment with green brown =Wilson's disease

386- Mother is concerned about her 5 month old baby that he’s delayed developmentally how would you
relieve her concern ?
A. baby is waving hi
B. baby is setting independently"
$
#
C. baby is holding objects

387- congintal adrenal hyperplasia , now he is dehydrantion + slight low glucose what will give?
3thing ( normal salin + steroid + glucose )

389- High levels of 17-OH progesterone can indicate acongenital adrenal hyperplasia (CAH) =How to
manage ?
daily hydrocortisone orally

44
390- Female child has several episodes of vomiting and enlarged clitoris on examination. studies are given
and show sodium 120 with other electrolyte imbalance treatment = corticosteroids
start IV fluid
) congenital adrenal hyperplasia case
: ‫رﺑط‬
*C*ongintal = *c*litoris =*c*orteson

391- 2months old develop diarrhea without vomiting or any other symptoms The mother concerns about
dehydration management?
A. oral rehydration solution"
$
#
B. IV fluid

392- Child was taking oral rehydration solution, present with mild dehydration, what is the reason of his
symptoms?
A.lactose intolerance
B.fructose intolerance
C. glucose intolerance"$
#

393- Mother came with baby 12 months suffering from recurrent gastritis after introduce normal diet again
according to previous pediatrician give him oral rehydration Now baby came with same feature with mild
dehydration also =Oral rehydration for 24 day then give normal diet ( not for 5 days)

394- Pediatric patient with classical symptoms of DKA + elevated blood glucose . What will you do next?
Urinedipstick
= ( ‫ﻋﺷﺎﻧﻧﺷوﻓﺎﻟﻛﯾﺗون‬Dont choice Hba1c bez Q next step

395- Child lossing 1kg despite he eating and drinking a lot , came dehydrated and irritable What's the
diagnostic test to the reach the diagnosis=
urine dipstick"#
$ "
#$

396- Pediatric presented with hyperglycemia and sx of diabetes In vs RR:60 What is the most important
test to do: Hba1c 8 == Urinalysis ( to detect the keton).

397- Diabetic mother, how to know if baby will be normal? - HgA1C"


$
#

398- Child diagnosed with T1DM screen eyes how often =after 5 years then annually

400- When do screening for uveitis in SLE patients with (-) ANA. = 6m

401-child with type 1 DM, what he will use to control the disease? Regular insulin
Control = ‫ = ﺗﻧظﯾم‬regular

402- old DM type 1 complain of hypoglycemia best TTT ?


decrease Mixture insulin"
$
#

403- Child didn’t eat anything last 3 h came with severe thirst . Fasting blood glucose 6.3 . Why he is
increase of glucose ? decrease insulin

404- child on (glargine + aspar) complain of fasting + postprandial hypoglycemia :reduce both

405- DKA During the management most important thing to be monitored?cerebral edema

406- DKA treated but still have hypokalemia why ? ( vomting )


45
407- Child with dehydration, depressed anterior fontanel, and decreased skin turgor. What is the percentage
of dehydration?
А. 5.
В. 10 "
$.
#
С. 15.
D. 20
if asymptomatic> mild> 1-5%
if more > moderate> 6-10%
if severe (hypotension)> 15%
in children of diabetic mothers, glucose 12.5% is given in? A. central line

408- Newborn with hypoglycemia what is the route of 20% dexterous? Central line
= 10priphral = i = 10
= 12.5 + 20central

409- newborn with one umbilical artery , what’s the cause: mother with DM

410- mother complains her daughter is less than her age +parent or one of then short. all lab results normal
except insulin growth hormone is low = growth hormone defincy

411- Child 9yrs old came with his mother because she thinks he is short stature. mother is short. When hand
bone examiend revealed age of 7 years Investigation All normal including growth hormone .
Except insulin like growth hormone was 18 low, What is diagnosis?
.Growth hormone deficiency" $ constitutional ( not Growth hormone)
#

412- 2y/o developing breast which case ? premature thelarche

413- 7y old female started breast development, and pubic hair start to appear and acne = central"
$
#

414- 5yrs female with pubic hair , no clitoromegaly obese, hight above 90 centile?
Dehydroepiandrosterone Sulfatedehy DHEA

415- girl 7 years old has pubic hair, developed breast ,ask about which kind of puberty ?
•central precocious puberty"
$
#

416- Child with short stature...parents concerned whether he will remain short in the future as well?!
What is the most important thing in history that would determine your - parents hight"$
#

‫ﻟوﺟﺎءوﻗت‬puperty = ‫وﻟﺳﺔﻣﺎظﮭرﺑﺎﻗﯾﺎﻟﻌﻼﻣﺎﺗﻧﺧﺗﺎر‬constitutional

417- 9y/o boy, her mother concern about short stature, the investigation result the bone density for age 7
years= constitutional ( not Growth hormone)

418- male with type hair and dark secrtum = tanner stage 4

419- Boy with pubic hair towards adult distribution and darkening of scrotal skin. Tanner stage? A.II
B.IV"$
#

420- 13years old brought by her mother concerned about her stature, patient is normal, on examination, no
signs of breast development and no pubic hair, what is the cause? Constitutional

46
421- Mother came with her 9 years old complaining about his height. She's short (didn't specify how short),
Patient is 25kg and 120cm, His bone exams revealed that his actual bone age is 7 years, Diagnosis:
A. Constitutional."
$
#

422- Case of gonadal agenesls she was 17 years no period mInImal development of breast with axlllary and
pubic hair Outflow obstruction or mullarian agenesis = pelvic US

423- 17years old boy with unilateral gynecomastia: Reassure, it will disappear later

425- 7Y tanner stage 5 (breast, pubic hair,acne) type of puberty? Precocious puberty

426- 7year old with pubic hair, no axillary hair, no breast or mensis?
Precocious puberty ( Not adrenarche Bez adrena means > axillary and pubic hair and this case no axilla hair
)

427- 9days newborn come with jaundice only in the face not extended to the rest of the body.. otherwise he
is healthy was delivered by NVD with no completing.. and he was breastfeed immediately. what is the cause
of his jaundice ?
breastfeeding jundice

428- 2years old her mother noticed development of the breast no other sign of puberty
A. precocious puberty
B. premature puberty
C. premature breast development"$
#

429- 4months on breastfeeding, This is her first baby came with 2 days hx of lethargy constipation, fever,
response weak when light directed to his eyes , cause =
A. Hypothyroidism honey
B. Infantile botulism"
$
# ‫اكل‬
C. Guillain barre
D. Poliomyelitis

430- 25y/o primigravida , baby flat face , no smile:


A- infantile botulism"
$
#
B- congenital hypothyroidism

431- child has leg abnormal shape and delayed walk, ca high ,alkaline phosphatase high ,normal phosphate
?
A- rickets ."#
$ " vit D deficincy
#$
B- familial hypophosphatemia ricks .
C- renal dystrophy
D- hypophosphatemia

432- A child presented with lower leg long bones angulation. Labs show high CA and low phosphate. X ray
shows distal bone hypertrophy
A. rickets ."
$
#
B. familial rekits .
C. renal osteodystrophy

433- Baby delivered at home presented 65 days later with rt thigh bruises other exam unremarkable
)PT high , PTT high , other normal= (
hemorrhagic diseases of newborn
47
434- Pt 5 days old baby with thigh bruises, home delivery, lab : high pt and ptt Dx:
A. hemorrhagic disease of newborn" $ .
#
‫…ﺗﺷوﻓوﻧﺑﺎﻟﺳﯾﻧﺎرﯾواطﻔﻠﮭﺗوھﻣوﻟودوﺣﺻﻠﻠﮫ‬bruses‫اﺧﺗﺎروا‬A‫ﻣﻧﺎﺳﻣﮭﺎواﺿﺢ‬

435- Q/ in prolong delivery mother what complication to baby will have After use of ventous the injury not
cross the bone ==
Right answer *cephalohematoma*
Note: if the Q said = cross suture line then chose > Caput succedaeum
-

436- Indicator of cystic fibrosis= poor wait gain

437- cystic fibrosis , what u will find else in = Nasal polyp

438- On examination he is jaundice, has large fontanel, cold extremities + hypotonia + large toung to his
mouth. What’s likely diagnosis?congenital hypothyroidism " $
#

439- 2 month old mother notice bulging tongue , dry mouth, constipation , fhx of autoimmune dis , tSH high
22 , T3 low T4 low what mx ==. give life long levothyroxine. ‫ﯾﺎﺧذوﻧطواﻟﺣﯾﺎﺗﮭﻣﺛﺎﯾروﻛﺳﯾن‬

440- Newborn what is the SINGLE investigation you must to do? Thyroid function test
‫ ھﮭذااھﻣﺗﺣﻠﯾﻠﻧﻌﻣﻠﮫ‬. ‫ ﻻااازﻣﺎﯾﻣوﻟودﻧﻌﻣﻠﻠﮭوظﻌﺋﻔﺎﻟﻐدةﻻﻧﻠوﻛﺎﻧﻌﻧدھﺎﻧﺧﻔﺎﺿﻔﯾﺎﻟﺛﺎﯾروﯾدوﻣﺎﻟﺣﻘﻧﺎﻋﻠﯾﮭواﻋطﯾﻧﺎﻟﮭﻌﻼﺟراﺣﯾطﻠﻌﻣﺗﺧﻠﻔطوﻟﺣﯾﺎﺗﮫ‬.

441- Newborn developed jaundice in first 12 hours , labs Low hb “9” Which test to order= . Fragility
test‫ﻓراﺟﯾل = اوﻟﻣﺎﯾوﻟداﻟﻣوﻟودﻧﺟﻠﺳﻧﺗﻔرﺟﻌﻠﯾﮫ =ﻧﺗﻔرﺟﻔراﺟﯾل‬

442- positive direct and indirect comp test what is diagnosis = Autoimmune hemolytic anaemia ""

$$
##
‫ﺑﺳﻧﺷوﻓﻛﻠﻣﺔ‬poative comp
‫ ﻧدورﻋﻠﻰ‬hemolytic

443- 11 years old with with jaundice Lab test: Increase indirect bilirubin Increased total bilirubin Increase all
the lft ? gilbert
‫ ﻟوﺷوﻓﻧﺎﻛﻠﻣﺔ‬indirect billurupin ‫ ﻋﺎﻟﯾﻧدورﻋﻠﯨﻛﻠﻣﺔ‬Gilbert
‫ اﻧﺗﺑﮭوا‬indirect ‫ وﻟﯾس‬direct
444- What indicate hemolysis ? Increase unconjugated

48
445- 5d /o baby be jaundice asked about important q ask in Hx ?? blood group "
$ q
# u
t
s
r

spherocytosis =osmotic fragile

‫اھﻣﺷﻲءﻓﻲ‬billary atresia ‫ا‬


‫ ﻧﮭﯾﺣﺻﻠﻠﮫ‬jundice ‫ اوﻟﻣﺎﯾوﻟدوﻣﺎﯾروﺣﺑﺎل‬photophopia ‫ھذااھﻣﻛﻠﻣﺔ‬
‫وال‬
Direct and total biluropin
‫ﻋﺎﻟﻲ‬

446- Pedia 4 days with jaundice and his brother had the same thing ((Direct bilirubin and total bilirubin))
was high = biliary atresia

447- child was delivered, and he developed jaundice on a 3rd day. He was treated for physiological jaundice
but 2 weeks later jaundice became progressive with associated pale stool. Investigations
done and showed: Total bilirubin high, Direct bilirubin high. What's the diagnosis= Biliary atresia ""
$$
##

448- target cells and inclusion bodies, dx= SCA


‫ = رﺑط = ﺳﯾﻛل = ﺳﯾﻛﻣرﯾض = ﻟﻣﺎاﻟواﺣدﯾﻣرﺿﻧﻛوﻧﻛﻠﻧﺎﺣوﻻﻟﻣرﯾض‬inclustion body

449- SCD patient came with crises, What’s the appropriate next step to reach diagnosis (to differentiate
between aplastic and splenic sequestration?A. Reticulocytes

450- SCA medication will prevent acute chest syndrome)hydroxyurea

451- Pic of CXR of rt lobe consolidation With long scenario of SCD pt presented with ))))chest pain((((( and
dyspnea and back pain, What is the Dx= Acute chest syndrome

452- SCD young patient came to the ER with toxic looking, enlarged liver and spleen, drop in Hgb=
sequestration crisis

453- Child with SCD most Important long tami treatment: Hydroxyurea

454- Child with sickle cell anemia presented with shortness of breath and chest pain on
is the best initial step in A.lv fluid and analgesic "
$
#

456- 6yr Sickler with fatigability for 2days. found anemic Spleen 6 Cm below costal margin = Splenectomy

’ Hg A2 higher in minor b thalassemia




’HgF in major thalassemia


‫ رﺑط = ﺣرف‬F = ‫= ﻛﺎرﺛﺔﻛﺑﯾرة‬major

457- Case of child studies given microcytic anemia he has high Hba2 on electrophoresis what type of
anemia
Beta thalassemia minor

458- Child lab showed (((microcytic and hypochoromic anemia )))with(( reticulocyte count high (2%) ,
ferritin normal, and his ((2 siblings ))have the same presentation what is the diagnosis ?
A. Sickle cell disease sisters
B. IDA

49
C. Alpha Thalassemia trait """
$$$
###

459- 58 days baby have G6PD they give lab value low HB level : what are the causes of this:==hemolytic
anemia
High RBC low HB Low MVC Normal Reticulocyte= Anemia of chronic disease
hemarthrosis =Hemophilia

460- ring-shaped/headphone-shaped trophozoites are seen in case of Malaria = Plasmodium falciparum


infection
‫ ﺣرف‬F ‫ = ﻛﺎﻧﮭﺎﺻﺑﻌﻔﯾﮭﺧﺎﺗم‬falciparum

461- microspherocytes ++ Anisocytosis =hereditary spherocytosis =‫ ﻣﻧﺎﺳﻣﮭﺎﻗﺎﻟﻛم‬microspherocyte .

(rocker foot= Edward


Child eats alot and he is obese with undecended testis , facial malformation and cleft plalte? pradder willi
syndrom

462- marfan =Tall short neack +ammnehria = turner


463- Down syndrome endocrine association = Hypothyroidism =
‫ = داون = اﺳﻔل‬hypo

464- Down syndrome =AVSD ‫ارﺑﻌﺣروﻓﻣﻌﺎرﺑﻌﺣروف‬

465- down syndrome?Trisomy 21

466- Low incidence in down syndrome: Mosaicism


‫“ﻧﻘﺷراﻟﻣوزﻻﺳﻔل‬
—‫ رﺑط = ﻣو ز‬dowen


467- Mother 27 years I think had Down syndrome baby what Increases hor risk for
having another baby with Down syndrome =
A. age "
$
#
B. father chromosome
‫ ﺳﻧﺔﯾزﯾداﺣﺗﻣﺎﻟﯾﺔ‬٤٠ ‫ ﻓوﻗﻌﻣر‬down

468- coarctation of the aorta associated with a. Turner "


$
#

50
469- Infant months of age died , they took hx from the parents ( the infant was preterm, problem with lungs ,
parents are heavy smokers and he was sleep with them ) what is the cause of death ?== sudden death infant
syndrome ""
$$
##
‫اﯾﻣرةﺑﺎﻟﺳﯾﻧﺎرﯾواﯾﻘوﻟﻛطﻔﻠﻣﺎﺗﺑدوﻧﺎﯾﺳﺑﺑوﻻﺳﺑب‬
‫ دورواﻋﻠﯨﻛﻠﻣﺗﯾن‬:
Smoking ‫اﺑوھﯾدﺧﻧﻘرﯾﺑﻣﻧﺎﻟرﺿﯾﻌﻔﻣﺎﺗﺑﺳﺑﺑﺎﻟﺗدﺧﯾن‬
‫ اﻟﻛﻠﻣﺔاﻟﺛﺎﻧﯾﺔ‬:
Social hx
‫ﻟﻛﻧﯾﻘوﻟﻛﺳوﺷﯾﺎﻟﮭﺳﺗورﯾﯾﻌﻧﯾﺗﺳﺎﻻﻻﺑﺎذاﻣدﺧﻧﺎوﻻ‬،‫ﻣﻣﻛﻧﺑﺎﻻﺧﺗﯾﺎراﺗﻣﺎﯾﻘوﻟﻛﺷﻲءﻋﻧﺎﻟﺗدﺧﯾن‬

470- turner=fold skin at nape of neck

471- Malnutrition of african boy with Acitis or cetral edema A. Kwashiorkor = ‫رﺑط = ﻛواﺷﻛﯾر = ﻛرش‬
‫ﯾﻛوﻧﺑطﻧﮭﻣﻣرةﻛﺑﯾرة‬
brittle hair and abdominal destination diagnosed with kwashiorkor cause? Protein malnutrition

472- Malnutrition of african boy with Atrophy everywhere = marasmus

473- mumps: parotid ""



$$
##
‫ ﻣﺎم‬: ‫ رﺑط‬mum = ‫اﻛل = ﺑﺎروﺗﯾدﻗرﯾﺑﻣﻧﺎﻻﺳﻧﺎﻧﻠﻠﮭﺿم‬

374- 4 year old boy most common cancer == leukemias

475- child has swelling and redness above knee ( femur)=osteosarcoma

476- Orthopedic tumor in pedia there is No limitation of movement? osteosarcoma

477- Pt diagnosed with small cell lung cance, presented with dehydration, serum osmo low, urine osmo
high. ‫ ھذھﺎﻋراﺿﺎﻟﺟﻔﺎف‬.
Ttt: Normal saline

475- Flank *mass* on children = wil*ms* (nephroblastoma )

476- Child presented to the ER with fever and abdominal pain. After a fall 1 day ago, the mother noticed
abdominal distention mainly on the right side. On examination he is ((pale and hypertensive(( , your dx ?
.A Pyelonephritis
B. Wilm's tumor.
C. Neuroblastoma " $
#
if central = neuroblastoma If flank = Wilms tumor

478- Flank mass 1st Q what is the investigation: CT

479- Boy came to ER has erosions bone in the hand and in the lab has high Ca, phosphate What the
treatment you will give ? Hydration + biophosphanate‫ﻋﺷﺎﻧﻧطﻠﻌﺎﻟﻛﺎﻟﺳﯾوﻣﻧﻌطﮭﻣﻧورﻣﺎﻟﺳﺎﻟﯾﯾن‬

• Milestone ‫ ﻣﻌرﺑطﮭﺎ‬....
480- *1-A child Can't sit without support, but he can coos and laughs. Estimated age is ?* 3 month
‫وزﯾﺎدةﻋﻠﯾﮫ‬. ‫ ﺿﺣك = ﺷﻔﺗﯾﻧﻔوﻗوﺗﺣت = ﺷﮭرﯾن‬coos ‫ ﺷﮭور‬٣ ‫ﯾﺳﯾر‬

481- *tells a story, hop on one leg, The estimated age is ?*


A. 4-5 years
51
B. 3 years
C. 6-7 years
D. 5-6 years
Hop on one leg = 4 years
‫اﻟﻘدﻣﺎﻟطوﯾﻠﺔاﻟوﺣدةﺗﺧﯾﻠواﺷﻛﻠﮭﺎﺗﺗﻛوﻧﻣﻧﺎرﺑﻌﺎﺷﯾﺎء‬
‫ ﻗدم = ارﺑﻌﺳﻧواﺗوﻟﯾﺳﺷﮭور‬+ ‫ ﺳﺎق‬+ ‫ رﻛﺑﺔ‬+ ‫ﻓﺧذ‬
Thigh + leg + foot + knee

482- baby says "baba", walks holding furniture and Walks with two hands held, The estimated age is ?*
A. 8 months
B. 4 months
C. 2 months
D. 10 months "
$
#
‫= رﺑط‬
Walks with two hands
‫ ﺷﮭور‬10 = ‫ﻋﺷراﺻﺎﺑﻊ = اﻟﯾدﯾﻧﻌددھﻣﻌﺷراﺻﺎﺑﻊ‬

483- A child that can raise his head slightly when prone and smiles. He turns his head 180 degrees and has
no head lag when you pull him to sit. How old is he?*
A. 4 weeks
B. 8 weeks
C. 12 weeks
D. 16 weeks "$
#
‫ اﺳﺑوع‬16 ‫ = ﯾﺳﯾر‬160 ‫ ﻛﺎﻧﮭﺎﻗرﯾﺑﺔﻣن‬180 ‫اذاﺷوﻓﺗوا‬

484- Child can set without support, cruises around furniture, (uses chair to sit) says dada, crawls well. What
is the age of this child ?*
A. 8 months
B. 10 months✅
C. 12 months
D. Non of above
10 ‫ وﺷﺧﺻﺟﺎﻟﺳﻌﻠﯾﮭظﮭرھواﻟﻣﻘﻌدﯾﺳﯾرﻣﺟﻣوﻋﮭم‬+ ‫ﺗﺧﯾﻠوااﻟﻛرﺳﻲ‬
Chair = 10

52
485- A milestoe of a baby can say baba and walk (holding furniture) ?*
A. 6 months
B. 8 months
C. 14 months
D. 10 months "
$
#
‫ﯾﻣﺳﻛﺎﻻﺛﺎﺛوھوﯾﻣﺷﻲ = ﺗذﻛروااﻟﻛرﺳﯾﻣﻧﺿﻣﻧﺎﻻﺛﺎث‬
.
486- Baby setting in mother’s lap unsupported, when the doctor spoke the baby turned around and laughed
and babbled to the doctor, baby’s age?
A. 7 months "
$
#
B. 5 months
C. 9 months
D. 2 months

‫ طﻔﻠﺟﺎﻟﺳﺑﺣﺿﻧﺎﻣﮭواﻟدﻛﺗورﯾﺗﻛﻠﻣﯾﻧظرﻟﮭﺎﻟطﻔﻠوﯾﺿﺣك‬.
setting + lap + mother +Doctur + laugh + spok+ babbled = 7

487- (Baby can sit without support ) + roll from prone to supine and back play and handle objects but can't
pick things between 2 fingers, What is the diagnosis ?*
A. 4 months
B. 6 months "
$
#
C. 18 months
D. 9 months
Baby can sit without support =
‫ ﻛﺎﻧﮭواﺣدﺟﺎﻟﺳوظﮭرھﻣﺳﺗﻘﯾم‬٦ ‫ ﺑﺎﻟﻌرﺑﯾﺎﻗﻠﺑوارﻗم‬٦ ‫ﺗﺧﯾﻠوارﻗم‬

488- Which of the following is related to the "gross motor" developmental milestone of a
(6 -month- old infant) ?*
A. Sits unsupported "
$
#
B. Rolls from prone to supine
C. Rolls from supine to prone
D. Walks with suppor
‫ﻧﻔﺳﺎﻟﺳؤوال‬

490- Which of the following is one of the expressive language developmental milestone of a 3 -year- old
boy ?*
A. uses 2 words sentence
B. uses 3 words sentence "
$
#
C. uses feeling words
D. tells stories
‫ﻣﻧﺎﺳﻣﮭﺎ‬
3 years = 3 sentence

491- A concerned mother brought her child for a routine evaluation. She is concerned that the child’s
development may be delayed. Regarding developmental milestones, which of the following is an appropriate
match?*
A. Drinks in a cup, walks without support – 18 months "$
#
B. Sits without support, roll from back to tummy – 4 months
C. Stands with support and plays “peek-aboo” – 6 months
D. Stands without support, cruises around furniture – 9 months
(walks without support – 18 month)
‫ ﺑﺎﻟﻌرﺑﻲ = رﺟﻠﯾﻧﻛﺎﻧﮭﯾﻣﺷﯾﻌﻠﯨرﺟﻠﯾﮭﺑدوﻧﻣﺎﯾﻣﺳﻛﺎﺣد‬٨ ‫رﻗم‬

53
492- A mother brought her 1-year-old son to the doctor for regular check-up. Considering the child has
normal growth and development, which of the following should this child not be able to do?*
A. Wave “bye-bye”
B. Say “mama” and “dada”
C. Walk holding on to furniture
D. Stand on tip toes "
$
#

493- A child can walk without support, tries climbing on furniture, build 3 cubes and (points to something
he is interested in) , How old is the child?*
A. 15 months " $
#
B. 24 months
C. 6 months
D. 9 months
point to something he is interested in =Point =5 = ‫ﺧﻣﺳﺣروف‬

494- At what age does the infant (head lag disappear )=


A. 1 month
B. 4 months "
$
#
C. 6 months
D. 12 months
Head = 4 ‫ﺣروف‬

495-4 months old boy what you can expect to his age?
A-try to crawl.
B-smile.
C-sit without support.
D- fix his head ""

$$
##

496- A child ride tricycle but can't draw a square, what is his age?*
A. 2 years
B. 3 years
C. 5 years
D. 6 years
tricycle = 3

497- What age does a normal baby should smile at?*


A. 1 month
B. 2 months "
$
#
C. 3 months
D. 4 months
2 = ‫اﺑﺗﺳﺎﻣﺔ = ﺷﻔﺗﯾﻧﻔوﻗوﺗﺣت‬

498- Smile but not reach objects?. 8weaks "


$
#

499- What is the milestone of a baby that can( hold his head) and when he looks at his flying hands he
laughs and coos ?*
A. 4 months "
$
#
B. 18 months
C. 10 months
D. 3 montht
hold his head = ‫ = ﯾﻌﻧﻲ‬head lag disappear

54
‫واﺗﻔﻘﻧﺎان‬head lag diapear ‫ ﺷﮭور‬4 ‫ﯾﺧﺗﻔﯾﻔﯾﻌﻣر‬
Head = 4 ‫ارﺑﻌﺣروف‬

500- A mother came with her child. Her (child walked into the clinic.) He can (say few words.) He mimics
his mother and Plays with a doll. what is the most likely age of this child?*
A. 12 months
B. 15 month "
$
#
Clinic = 5 ‫ﺣروف‬

501-At which age baby can say few words


A. 12 months
B. 2 years
C. 18 months "
$
#
D. 6 months ( ‫رﺑﻂ‬few words = 8 ‫ﻋددﺣروﻓﮭﺎ‬

502- A child walks alone & Builds tower with 3-4 blocks, his estimated age is ?
A. 24 months
B. 18 months "
$
#
C. 10 months
D. 14 months
(3-4 blocks)
3 × 4 = 12 + blocks = 18
‫ ﻋددﺣروف‬blocks = 6
6+ 12 = 18 :)

503- (baby pulls himself to stand) , crawls without difficulty, which age is considered ?*
A. 8 months
B. 9 months "
$
#
C. 10 months
D. 11 months
crawls = ٩ ‫ = ﯾﺷﺑﮭرﻗم‬crawls ‫ ﺑﺳﻧﯾﻣوااﻟرﻗم‬٩ ‫وھوﺟﺎﻟﺳﯾﺣﺑﯾﻘرﯾﺑﻣﻧرﻗم‬

504- Roll from prone to supine and vise versa + grab with only two fingers. What is the milestone?*
A. 2 Months
B. 4 Months
C. 8 Months
D. 12 Months✅
grab with only two fingers. = Tow = 2 = 12
‫ اﻛﯾدﻣﺎﯾﻘدرﯾﻌﻣل‬grabs 12 = ‫ ﺑﻌﻣراﻟﺷﮭرﯾﻧﻌﺷﺎﻧﻼﺗﺧرﺑطوا‬month

505- Child that throws a ball at you and draws a straight line and stacks “few” cubes on each other (they
didn’t mention the number of cubes). What is the age?*
A. 12 months
B. 14 months
C. 18 months
D. 24 months "$ (2 years)
#
straight line =
‫ﯾرﺳﻣﺧطﻣﺳﺗﻘﯾم‬
2 = ‫ﻟﻣﺎﻧرﺳﻣﺧطﻣﺳﺗﻘﯾﻧﯾﻛوﻧﻠﮭﻧﻘطﺔﺑداﯾﺔوﻧﻘطﺔﻧﮭﺎﯾﺔﯾﺳﯾرﻧﻘطﺗﯾن‬
‫ ﯾﻌﻧﯾﺳﻧﺗﯾن‬.
506- Baby can tell u complete story = 4 years .
‫ﻗﺻﺔﻛﺎﻣﻠﺔ = ارﺑﻌﺳﻧوات‬

507- Rooting reflex when disappearing?


55
4m
root= 4

508- Same as this pic ask which reflex. rooting

509- Pic of baby and physician hand. He was taping the left side of mouth When the primitive reflexes
disappear?4m

ste*pp*ing reflex ==disapper at 2 month


‫ ﺣرﻓﯾن‬PP = 2

510- what’s the time to say fever of unknown origin in pedia, per day :
A. 7.
B. 14 "
$
#
C. 21
D. 28
‫ = رﺑط‬unknown = ١٤ =٢ ‫ﺳﺑﻌﺣروﻓﻧﺿرﺑﮭﺎب‬
511- All reflex disappears after 4-6 months except stePPing at 2 months.
age the sucking reflex disappear? 6 m
٦ = ‫ ﻓﻣﮫ‬+ ٥ ‫ﻋدداﺻﺎﺑﻌﺎﻟﯾد‬

512- Child enters the dr’s clinic says “Hi”, mother gives him doll. Then he feeds the doll with milk bottle.
Mother moves his head then he says “No”. Mother does something and then he imitates her What’s the age?
A. 12 month.
B. 15 month.
C. 18 month " $.
#
D. 24 month
‫ ﺑﺳﺗﺷوﻓواﻛﻠﻣﺔ‬HI 18 ‫ﺣطوھﺎ‬
HI = 18

513- 8 Child feed the doll but can't us spoon what his age ?
A. 12 months
B. 15months
C. 18 months "$
#
D. 24
‫ ﯾﺑدءﯾﺳﺗﻌﻣل‬spoon ‫ﻓﯾﻌﻣراﻟﺳﻧﺔ‬
514- can sit in his own, stand alone, walk by the furniture, say word of repetitive consonant sounds like
"dada" can hold objects between his thumb and index finger = 10 month .
56
‫ رﺑط = ﻟﻣﺎﻧﺿﻣﺎﺻﺑﻊ‬thump and index ‫ ﯾظﮭرﻛﺎﻧﮭﺷﻛل‬O ‫وﻻﻧﮫ‬، stand alone 10 = ‫ﺻﻔرورﻗﻣواﺣد‬،‫ ﻛﺎﻧﮭرﻗﻣواﺣد‬.

516- Baby can walk and say 2 to 3 words rather than mama dada he crawls the stairs age? one year
‫ﯾﻘول = ﻣﺎﻣﺎ = ﺑﺎﺑﺎاوﻟﺷﻲءﺑﺎﻟﺣﯾﺎة = اوﻟﺳﻧﺔ‬

517- baby which is 4 years old and can hop on one leg but can't stay still for 10 seconds, can draw a square,
has a friend, can tell a story the mom is worried about the developmental milestones?
A. The baby is normal no need for referral ""
$$
##
B. He has a gross motor delay and fine motor delay
C. He has language and speech delay

518- 3 years old with normal milestones to his age except that he doesn't know how tospeak in future tense
and can't say 3-word sentence, what would you tell the mom
A. reassure.
B. he has a delay in speech only but other milestones are fine
Answer is: refer to ENT for audiogram

519- child with pincer grip but cant put pallets in a bottle. How old?
8 month

520- -Hemangioma in the left eye of a newborn and it needs to be resected so it doesn't affect the vision,
when will you do that?*
A. Immediately
B. 2 weeks
C. 6-8 weeks "
$
#
D. 6 months

57
521- Vaccination at age of school (6years) = Dtap , MMR, OPV, Varicella
‫ =رﯾط‬DTap = ‫ﺗﺎدﯾب = اﻟﻣدرﺳﺔﺗﺎدﺑﺎﻟطﻔل‬
‫ = ﻻزﻣﯾﺗﻌودﯾﺗرﻛﺎﻣﮫ‬MMR
Scho*o*l = *O*pv
Varicella = ‫ﻓﺎرﯾﺳﯾﻼ = ﯾﻼﺳﻠﻣﻘﺑﻠﻣﺎﺗروﺣﻠﻠﻣدرﺳﺔ‬

⚘⚘1 YO vaccination : ⚘⚘
OPV, MMR, PCV, varicella
‫ ھﻧﺎﻧﻔﺳﻌﻣراﻟﻣدرﺳﺔﻟﻛﻧﺑدل‬Dtab ‫ﺗﺎدﯾب‬
‫ ﯾﻛون‬Pcv

522- vaccine to give in 2mo old baby?


IPV hbv HIB DtaP PCV rota
‫ ﻛﻠﮭﻣﻣﺎﻋدا‬MMR

523- 4 months old vaccine =


IPV, HBV, Hib, DTaP, PCV
‫اﻻرﺑﻌﺷﮭورﻧﻔﺳﺷﮭرﯾن‬

58
524- Growth chart (similar to the pic) showing normal at birth, increasing but below the 3rd centile,
Diagnosis? failure to thrive

525- +ve cover test Diagnosis= Strabismus )cover= ‫)ﺳﺗر‬

henoch-schonlein purpura

diaphragmatic hernia
‫= اھﻣﺎھﻣﺷﯾﻔﯾﮭﺎاﻧﮭﺎوﻟﺧطوةﻧﻌﻣل‬NGT ‫ﺣﺗﯨﺑﺎﻟﺻورةاﻟﺧطﺎﻻﺑﯾﺿﮭذاﺗﯾوب‬

• Testicular Disease
526- Neonate ,(( palpable mobile mass non tender)) positive transillumination , what is it ?
A.Hydrocele "
$
#

59
B. Varicocele
‫ = رﺑط‬hydro = ‫ھﯾدرواﻣﻌﻧﺎھﺎﻣﺎء = اﻟﻣﺎءﻣﺎﯾﺳﺑﺑﺎﻟم‬

527- testicular asymmetry and was tender on palpation = testicular torsion = A testicle that's positioned
higher than normal or at an unusual angle.

528- 5 years old child found to have one testes in the scrotum and the other in the ingunal area what to do?
A-orchiopexy "
$.
#

529- Baby with painless scrotal swelling bilaterally , neg transillumination test, Dx?
A. Testicular torsion
B. Orchitis
C. ((Idiopathic scrotal edema))
‫ ﻻﻧﻘﺎﻟك‬painless ‫ وﻛﻼﻻﺧﺗﯾﺎراﺗﺗﺳﺑب‬pain

530- 15 years old with scrotal pain and absent cremasteric reflex diagnosis= tusticular torsion "
$
#

531- 12 years old male with testicular pain (horizontally lying, mildly elevated, no swelling or
erythema of scrotum) what to do next: -surgical exploration "
$‫ اﺳﺗﻛﺷﺎف‬-rehydration
#
‫ﻟﻧﻧﻧﺗظرﻋﺷﺎﻧﯾﺣﺻﻠﻠﮭﺎﻏرﻏرﯾﻧﺎﻋﻠﯨطوﻟﻣﻧﺎﻻوﻟﺟراﺣﺔﺑﺳرﻋﺔ‬

532- Child with bilateral inguinal hernia what is the treatment :


A. Herniotomy . "
$( bez child)
#
B. laparoscopic mesh repair

Child with impetigo (honey crusted lesion)?


- not contagious can go to nursery
- not contagious can go to nursery after 5 days
- contagious can go to nursery after 3 days
- contagious can go to nursery after lesion dry

Answer : D

Impetigo

533- Child with edema in eyelid and ankle with pale and dark urine . What you need in hx ?
A-impetigo in last month "
$
#

534- An infant presents with a 2 day hx of lethargy, malaise, and dark urine. What is tho Important thing in
hx that you must ask?
A. Recurrent UTls
B. Recent impetigo contagiosa "
$
#

535- Pedia present with case of impetigo ask when can return to day care:
D. contagious return when it gets dry.
B. not contagious return after 3 days "
$
#

536-Mom comes with her boy who had body rash for 2 days which was eruptions then became vesicular
then crusted with yellow secretions, asks when she can let him go to nursery? What you tell here?
C. the vesicles secretions are contagious and she can let him go to nursery when they get dry
60
537- Boy has a cat developed itching for a month with red eye and watery with discharge no
lymphadenopathy and general exam normal ?
A. Cat scratch.
B. Dermatitis allergic "
$
#
C. Conjunctivitis
D. Seborrheic

538- Baby developed truncal rash= Reassure

539- kid at party had (symptoms and sign of allergy) what cause? food allergy

540- Kid with stridor and rash on birthday = allergy food

541- neonate + less than one year common area eczema= * scalp *

542- Child more than one year area and Adults common area eczema=== - *Flexors* -

543- Most common place of psoriasis in childhood? A-Scalp "


$
#

544- child has eczema topical steroid i think now eczema get worse what next?
Parent Complince medication

545- Neonatal needs D10 fluids.. what’s the dose/kg? 2 ml/kgchild5= ‫ﺧﻣﺳﺣروف‬ml/kg for children

546-Fetal- bradykardia with sinusoidal what causes =anemia

547- after slnusitis surgery affect sensation in lower eye and upper lip nerve ? Infra-orbital

548- Baby 6 weeks, direct bilirubin high. Dx?


A- Crlglar najjar
B-ABO
C- Rh Incompatibility
D- Choledochal cyst "
$
#

549- child w/ gastroenteritis and they asked diagnostic test = Stool antigen "
$
#

550- Scenario about baby fall down on his RIGHT abdomen from 1 day then develop abdominal pain and
fever= liver contusion

61
551- difficulty passing stool , on PR examination empty rectum == hirschsprung ==Teated by resection and
anastomoses, leveling colectomy

552- Patient came with gastroentrietis When start normal diet after the ORT( in pedia
A. After 24 h

553- How to decrease the risk of airbag injury in children younger than 12 years: Restrain to back seat

554- Child with his parents start to develop fever lethargy, parents mention 2 days before changing his eye
color to yellow to he became ictrus, which type of viral hepatitis he had:
A. Hepatitis A
B. Hepatitis B
C. Hepatitis C
D. Hepatitis D

555- Child with attacks of severe midline abdominal pain with facial pallor. Poor appetite .. hx 2 = sisters
with migraine abdominal migraine

556- When say this pain from organ ( organic pain) ? =Before awakening

557- A child presented with 3 days of vomiting and stooling. Other information were given. What will be
the electrolyte derangement?B. Hypochloremic matabolic alkalosis " $
#
(Laila group started conrect the answer from here 558. Kindly do not hesitate to contact me any time for
further editing or correct answer
558- gastric cancer what is of high diagnostic value? fasting gastrin level q
u
t
s
r

559- celiac diseas , indicates activity of the disease= Anti Tissue trans**gluta**minase Ab

560- 16 years old male with 3 years history of diarrhea ( food containing )= Anti endomysial antibodies "
$
#

561- Q was high risk illness baby, from whom do you take details ?
A.Father
B. Mother "
$.
#
C. House

562- Signs of obstruction in a child best initial modality of diagnosis =


A- US "
$.
#
C- chest x-ray
D- C- CT

563- Pediatric had gastroenteritis and she took metoclopramide that leads to involuntary movement facial
grimace and tongue protruding what to give
A. Diphenhydramine " $.
#
B. Epinephrine.
C. Cyproheptadin.
‫داﯾﻔﯾن‬diphen = ‫ دوﻟﻔﯾن = ﯾﻌﻣﻠﺣرﻛﺎﺗﻛﺛﯾرة = ﺗﺷﻧﺟﺎت‬.=
Metoclopramide side effect coz extra pyramidal symptoms, unvoluntray movement body, neck and face and
strange sensation in the bodh. First line ttt benzo larazebam or medazolam . second line Diphenhydramin

564- GERD CASE symptom increase at night only what to give her?Bed elevation & PPI
62
565- Case of pyloric stenosis : pylomyotomy

566- Child with projectile non biliary vomiting + mass in epigastric most accurate investigation?
US what treatment?pyloromyotomy

567-pyloric stenosis what occurs the diagnosis? - failure to thrive "


$
#

568- Pyloric stenosis what is electrolytes abnormalities:-hypochloremia hypokalemic metabolic alkalosis


Another scenario from glory 1&2
1-Long Case scenario of a Child with projectile non biliary vomiting
associated with failure to thrive. On examination you found epigastric
olive like mass. What electrolyte imbalance is expected to find in such
patient?
A- Hypomagnesemia
B- Hyperkalemia
C- Hypochloremic metabolic alkalosis ✨ ✅
D- Metabolic acidosis

2- Long Case scenario of a Child with projectile non biliary vomiting


associated with failure to thrive. On examination you found epigastric
olive like mass. What is the most appropriate management?
A- Pyleromyotomy . .
B- Gastrectomy
C- Pylerectomy✅
D- Colectomy and ileostomy

569- Child with Metronidazole and Omeprazole for eradication of H.Pylori drug you want to add =
Clarithromycin

570- liability of diagnosed necrotizing enterocolitis :


A.male sex.
B. pre- term "
$
#
C.young age
63
571- most important risk factor for Necrotizing enterocolitis: - birth weight < 1.5 kg "
$
#

572- NEC case asking about factor If its absent risk for NEC ?? Full term .

573- continuous bilious vomiting , Abd destention, Passes meconium after birth, now yellowish thin
diarrhea===.,Colic
a-hirschsprung disease
b-toxic megacolon
c-enterocolitis ""
$$
##
in this question the most dd for bilious vomiting is malrotation valvolus and second dd is necrotizing
enterocolitis. So it depends on age if first 2-3 weeks of life is necrotizing entero. If more than one month the
answer is malrotatiuon valvolus.

574- alkali causative ingestion =close observation


Same q from glory
Boy came with history of alkali causative ingestion with no symptoms.
What is the management:
A. Induce vomiting
B. put nasogastric tube
C. neutralization by water
D. close observation
Answer is: D

575- 9 years girl ingested two boxes (each 20 tablets) of paracetamol one dya ago after fight with her
mother? Mx: -
A- n-acetlcystine. –
B- astric lavage. –
C- observation and discharge pt ✅

٨ ‫ﻻﻧﺎوﻟوﺛﺎﻧﯾﺎﺧﺗﯾﺎرﺧﻼﺻﺧﻠﺻوﻗﺗﮭﺎﻟوﺟﺎﺗﺎﻗﺻﯨﺷﻲء‬
‫ﻓﺎھﻣﺷﻲءﻧﻧﺗﺑﮭﻠﻠﻌﻼﻣﺎﺗﺎﻟﺧﯾوي‬،‫ﻟوﺣﺎﺗﻔﯾﺎﻗﻠﻣﻧﺎرﺑﻌﺳﺎﻋﺎﺗﻘﺑﻠﻣﺎاﻟﻣﻌدةﺗﻔﺿﯾﻠﻼﻣﻌﺎءﻛﺎﻧﻣﻣﻛﻧﺎﻋﻣﻠﻠﮭﺎﻏﺳﯾﻠﻣﻌدةﺑﺳﻣﻼﺻﻔﺎﺗﺎﻻوان‬،‫ﺳﺎﻋﺎﺗﻛﺎﻧﻣﻣﻛﻧﺎﻋﻛﯾﮭﺎاﻟدواء‬
‫ةوﻧطﻠﻌﮭﺎﻟﻠﺑﯾت‬
In this question the right answer is A as long as pt consume 20 tablet so high dose even after 24 hours.
64
I will not change till you double check and get back to me for editing ( Laila)

576- Child ingested iron Serum iron 90 Came with nausea and vomiting I think 9 hours ago, Mx?
A. Gastric lavage
B. IV deferoxamine "
$
#

578- Paracetamol poisoning with symptoms ttt? N-acetyl salicylate

579- adult polycystic kidney asking about screening for him=. Ultrasound abd

580- high potassium unresolving to treatment) =Ca-gluconate


In this question we only give ca gluconate if there is any change in ECG stabilize the heart.
Usually we use in hyperkalemia insulin, Ventolin and dialysis. So in the question if he said pt not responed
to ttt it means he take insulin and Ventolin so the next step is dialysis

581- child developed generalized edema with fever and dark urine, all labs normal except low calcium and
low albumin, what is dx?
A. Minimal change disease "
$
#
B. Mesangial Disease
C. Nephropathy
D. May be Glomerulonephritis

582- Child e Nephrotic syndrome What medication to avoid in management:


A. aspirin
B. amoxicillin
C. acetaminophen
D. ibuprofen "
$ this is the only choice affect the kidney the rest metabolize in liver this is in
#
nephrotic syndrome. Compare with reys syndrome the pt should avoide aspirin. So the answer he
is D

583- Child came lethargic ,fever,abd pain hls family say they have history of )) mldotrlnlan disease((
Mediterranean fever ,Labs
normal except ratlculocytes high which of tho following drug contralndlcatad = Amoxciline
aspirin .
65
This pt has meditarian anemia so the ttt is colchicine

584- when you said that child have steroid resistant =Induce remission in 4 weeks

585- Child with nephrotic (diagnoses given), after ttt given when can you confirm the baby completely
resolve?
A. After normal dipstick 3 Consecutive time.))-urine analsyis negative for protien for 3 days((

So in this question and answer is normal 3 dipstick not urin analysis however douple check

66
586-child with facial and lower limb edma with protinurea on thiasid and other medication i don’t
remember wt best next step?- switch thiaside to fursmide "
$ q
# u
t
s
r

587- Pediatric complain of arthralgia, SOB, rash in lower extremity , fever past hx of dental procedure
before 2 month and sore throat before 2 week . on lap proteinuria and hematuria , On examination murmur ..
what diagnosis
A. infective endocarditis.
B. post- streptococcal infection """

$$$
### so post strepto comes with glomerulonephritis so pt came
with hematuria and comes with infective endocarditis so pt came with murmur

588- Child has abdominal mass, aniridia and undescended testicle =wagner disease
Wagr syndrome
•aniredia
•genitourinary anomliey
•wilims tumor
•mental retardation
Wegner disease
•necrotizing vasculitis
•necrotizing granuloma in lung
•necrotizing glomulonephritis

589- Child with glomerulonephritis then develop hemoptysis ? goodpasture syndrome

590- Asymptomatic hematuria in a child Urinanalysis :+ve RBC What to do next ?


A. Repeat urinalysis "
$
#

591- Child presents with hematuria. history of constipation for 1 week and presents with hematuria and
urinary incontinence. Upon examination he had tender abdomen, Labs: Low Hb, RBC in urine, Protein in
urine, WBC in urine == UTI (not PSGN)
67
So UTI can comes with any urinary symptoms: dysuria incontinence retention hemtouria. In this key words
constipation and WBC. IF HE SAID WBC CAST IT IS PSGN

592- Case of uti and asks what go with urinary tract infection as a risk factor: Fever

593- This child with same condition group of children in school .. (i suggest its chicken box)
A. sole spot
B. skin scratch✅
C. tonsillitis

594- Case about a child with increased water consumption and going to the toilet, not to mention Urine and
serum osmolality, not mention any electrolyte, with no response to water deprivation test, no other
complain:
A. central DI
B. peripheral DI
C. SIADH
D. drinking too much water. "" $$
## q u
t
s
r
In this question we are not 100 % sure about the answer ‫اﻟﻣﺷﻛﻠﺔ ﺑﺎﻟﺧﯾﺎرات اﻟﺑﺎﻗﯾﺔ اﻧﮫ ﻛﻠﮭﺎ ﺗﺄﺛر ﻋﻠﻰ ﻧﺗﺎﯾﺞ اﻟﺗﺣﺎﻟﯾل‬
‫ ﺑس اﺗﻔق ﻣﻌك ﺑﺎﻟﻧﻘطﺔ اﻟﻠﻲ‬.. ‫ ﻋﺷﺎن ﻛذا اﺳﺗﺑﻌدت اﻟﺧﯾﺎرات اﻟﺑﺎﻗﯾﺔ‬.. (‫ اذا ﻣﺎ ذﻛر ﻧﺗﺎﯾﺞ اﻟﺗﺣﺎﻟﯾل ﺑﺎﻟﺳوال ﻣﻌﻧﺎھﺎ اﻧﮭﺎ طﺑﯾﻌﯾﺔ )اﻟﻣﻔروض‬..
‫ﻗﻠﺗﯾﮫ‬. Another issue if the answer D do you think sychogenic comes in this age group? Any one
explain

68
595- bedwetting =positive re enforcement and alarm

596- Bedwetting is considered normal until age: A. 5

597- Mother came with her son, and concern about recurrent bed wetting, the treatment effect will be on
which muscle: A. Gluteus B. Perianal C. Pelvic Floor "
$ D. detrusor muscle
#
IF not D option in exam go with C
598- Girl 8 yrs old with bald area on head. Mother says she keeps picking her hair due to exams and stress.
(trichotillomania) What medication will u give?
a. Olanzapine
b. Fluoxetine "
$
#

599- Child with symptoms of ADHD( Attention Deficit Hyperactivity Disorder ) interrupting the class
room and neighbor what is the treatment= dexmethylphenidate
if the Child 6 years and older is methylphenidate
If less than 6 years CBT

600- attention deficit hyperactivity disorder treat by = Methylphenidate q


u
t
s
r

601- Most Child fabrication symptoms by


A. Mother "
$
#
B. Father
C. Step mother
D. Care provider

602- First sign of female puberty Thelarche

603- first sign of male puberty?scrotum enlargement "


$
#

604- What is the problem of a 12 old child response to her parent's illness according to her developmental
stage:
A. difficulty to express emotions
B. Refractory behaviour: fussing, violences "
$
#
C. Imitating her parents symptoms when she plays
D. Does not understand their illnesses nature, cause, diagnosis, treatment

605- weeks old baby with strong cough and 2 episodes of him losing consciousness. on exam there is
intercostal retractions. ((02 sat was 90 )). What do you do?
Ventilator support ((not need bz ((02 sat was 90 )) D. B2 agonist " $
#
In this question the children before 2 years u cannot say this child has asthma. The dd before 2 years is
bronchitis so no need Ventolin or salbutamol. So if there is option oxygen admition observation choose it

607- Horizonal line in x ray of infant = Transient tachypnea of newborn

69
608- having tachypnea and grunting‫ﺷﺧﯾر‬most common cause?= RDS

609- Baby with x ray show ( TTN ) + symptoms of pneumonia + Lap high Neutrophils Ttt= Oral amoxicillin
7 day "
$ coz bacterial
#

610- ingested 20 tablet baby aspirin = . Respiratory alkalosis > metabolic acidosis. """
$$$
###
= ‫اﻛﻼﺳﺑرﯾن‬
‫اﻟﻛﺎﻟوز‬

611- A 2-hr old baby = What intervention would minimize disability in the first 6 hours?
. Mild hypothermia

612- presents with meconium aspiration =nitric oxide

613- 3m baby with respiratory distress when to admit for non invasive ventolatlon =PICU
Where to admit not when so the answer PICU

614- 18 m k/c of cerebral palsy presents with respiratory compromise Then put under MV Given ABG
,Pco2 normal,Po2 low ,PH 7.3 Ask what type of respiratory failure?
A. hypoxic
B. Hypercapnic resp failler type 2 "
$ = bith asphyxia
#
C. hypocapnic
Answer is:B since he is under MV

615- young girl presented with palpitation and hypercapnia. Father mentioned issues with schools and
exams. What is the diagnosis? Hyperventilation syndrome = school

617- Child with pneumonia hemolysis + positive agglutination test, what is the organisation=mycoplasma

618- Pre term baby diagnosed with hypoxic ischemic encephalopathy (HIE) what's the risk factor : Low
birth weight less than 1500 gm

619- Mother had obstructed and difficult labor, she gave birth to a child who wasn't crying and cynosed. His
ABG: pH 6.9, HC03 7, 02 8. What's the diagnosis? A. Hypoxic ischemic encephalopathy " $
#

620- 6y old child k/c of asthma came with asthma exacerbation , lab showed RR = 7/minutes , Hypercapnia
best initial ?
A. IV MG
B. Intubation "$ coz sever asthma pt need ICU and intupation
#
C. Albuterol
D. Theophylline

70
621- dyspnea attack initial management: Ventolin Neb

622- Pregnant lady, just delivered and she's known to have bronchial asthma. Which of the following
uterotonic medications you would avoid giving? carboprost F2 alpha ‫واﺣدﻋﻧدھﺎزاﻣﺔﻣﺎﯾﻧﻔﻌﻧﻌطﯾﮭﻛرﺑﯾﻛﻔﯾﺎﻟﻛرﺑﺎﻟﯾﮭﯾﻔﯾﮭﺎ‬
Agree

Contraindicated in severe cardiovascular, renal, and hepatic disease. It is also contraindicated in acute Pelvic
Inflammatory Disease. Hypersensitivity to carboprost or any of its components is also a
contraindication[2] Exert caution in asthmatic patients as carboprost may cause bronchospasm

623- 12 yo girl complaining of nausea and vomiting if the flight was more than 1 hour what is the best
anti*eme*tic for her ? D*imen*hydrinate
He mean Diphenhydramine ✅

624- Child presenting for check up clinic when to be concerned about growth ?
A-Body wt persisting at 10th percentile " $‫رﻗﻣﻌﺷرة‬
#
B-At 15th percentile
C-Progressing from 5th to

625- Criteria to help diagnose HTN in a 12 YO girl?


C. >90th percentile for age and sex
D. >95th percentile for age and sex "
$
#

626- During the evaluation of adolescent she was on the 95th percentile for age and sex, what of the
following action should be taken=Confirm size is more than average for age and sex

627- 3 years old boy his mom complained that she noticed blood spotting in the dipper,
there was another episode 3 months ago, no abdominal pain or other manifestation, on examination patient
looks pale and mucosal dryness, what’s the most likely diagnosis:
A. Juvenile polyps "
$
#
B. Mickle diverticulum = Mainly present with abdominal pain
C. Intusscesption = (red jelly-like stools), sometimes mixed with mucus.)

628- juvenile idiopathic arthritis case : both knees arthritis , nothing else, type?Oligoarthritis l
p
o
n
m

629- 12 y boy with right knee and wrist swelling and subcutaneous nodule = Juvenile rheumatoid arthritis

630- pedia 3y pt have painless bleeding per rectum ?


A. Juvenile polyp
Age 3 years or more and painless bleeding = juvenile polyp
2 years or less and abdominal pain wih bleeding = Mickel Diverticulum

631- Pediatric Patient Dx with cystic fibrosis, sibling no hx What appropriate management?
A-test siblings‫ اﻗرﺑﺎء‬for chloride sweat test.✅
B-test parents for chloride sweat test.
C-genetic test for CFTR gene for the sibling
D-genetic test for CFTR gene for parents

71
632- Pediatric patient presented with sudden apnea, has a history of upper respiratory infections, on
examination the patient takes coughs several hacking cough then takes a breath with inspiratory wheeze,
between coughing she looks fatigued and exhausted. No fever. Best investigation
A. Sweet chloride text "
$
#
B. CXR
C. ABG

633- A child presented with joint pain and hematuria. Mother reported history of URTI 4 weeks ago. On
exam there was petechial rash involving buttocks and thigh, otherwise normal. Platelets normal. What is the
appropriate Diagnosia? Henoch-Schonlein purpura
‫ھذااﻟﻣرﺿﯾﺗﻛوﻧﻣﻧﺛﻼﺛﺎﻋراﺿﺎﺳﺎﺳﯾﺔﺗﯾﺟﯾﻣﻌﮫ‬
Joint pain + abdominal pain + petecha
Ask about Treatment = mainly Suppurative. But with syptom active = Steroid .

634- Child has bloody diarrhea after a week develop petechial rash + hematuria and low plt Pt and ptt
normal ?
A- hemolytic uremic syndrome. ✅.
B-TTP
C-ITP
D-DIC

635- HUS= It results in the destruction of blood platelets (cells involved in clotting), a low red blood cell
count (anemia) and kidney failure due to damage to the very small blood vessels of the kidneys.
Lab results: low platelets and high
creatinine level. What is the diagnosis?
A. ITP.
B. TTP.
C. HUS ""
$$.
##
The answer is C due to : anemia, thrombocytopenia, kidney injury, heamatochazia
HUS should be with this triad:
H hemolysis
U uremia
S stomachache ( abdominal pain)

636- Child with fever hematuria headache ask dx = TTP

637- Rx pf TTP? plasma exchange

72
638- Treatment of thrombotic thrombocytopenic purpura = mainly by Plasmapheresis with or without
steroid
TTP ttt plasmapheresis but if delay we will chose FFP

639- .7 years old child day 7 post appendectomy come with fever and bad general .condition but her in
mechanical ventilation . After that do ct contrast . The child bleed from lvlne trachea and wound site , What
is the Diagnosis ? DIC "
$
#
Bleeding from each side

640- Dehydration baby what to found= Low pulse pressure or low urine output.
In this question the right answer is low urine output because of dehydration there is increase in pulse.

641- Child previous URTI came with petechiae and abrasion Every thing is normal except platelets 15000
ask about ttt ==A. Splenectomy B. Platelets transfusions (ITP CASE) "
$ C. Ivig
#

642- Child came from a visit to Africa. neck stiffness . can not elevate head or limbs : EBV "$
#
The right answer her after long discussion is GBS and it comes with meng symptoms. So neck stifness is
manigities until approve otherwise , and commonest coz of maningities is GBS and Pneumonia, However
check
‫ = رﺑط‬ebv = ‫اﺑﻌدﻋﻧﺎﻓرﯾﻘﯾﺎ‬.
643- Child Herpes gingivostomatitis sx (lips, gums, tongue, palate vesicles) and he can’t feed orally. What
will you give him?
A. Antiviral (i'm not sure if they write acyclovir or antiviral)
B. IV fluid and antiviral ""
$$
## ‫ﻻزﻣﻔﻠوﯾدﻻﻧﻣﺎﻛﺎﻧﯾﻘدرﯾﺎﻛﻠﺑﺎﻟﻔﻣﻔﻼزﻣﺗﻐذﯾﺔوﻛﻣﺎﻧﺎﻧﺗﯾﻔﺎﯾرال‬

644- A young child presented with tonsillar ulcer and painful lesion in the back of her mouth and soft palate
, what the dx A. Herp angia

73
645- Child with fever 39 ,sore throats on examination there is white exudate over the tonsils most
appropriate step to reach the diagnosis ?
A. Throat culture
B. rapid antigen detection test "
$ gold standar coz sensitivity from 80 to 90 %
#
First we take rapid antigen if negative ok if positive we do culture

, ‫احلمدلله الذي هدانا لهذا وماكنا لنهتدي لوال أن هدانا الله‬


, ‫نسأل الله أن يساهم هذا امللف في حتقيقنا للنجاح و للدرجات العالية‬
‫كل الشكر لقلوري تيم و دكتورة أالء وجميع القائمني على ذلك من تنسيق أو تصحيح‬
. ‫ال تنسوهم من خالص الدعاء بالتوفيق‬
. ‫هذا والله ولي التوفيق‬

74

You might also like